Women's Health Exam III

Réussis tes devoirs et examens dès maintenant avec Quizwiz!

A new mother with a thyroid disorder has come for a lactation follow-up appointment. Which thyroid disorder is a contraindication for breastfeeding? A.) Hyperthyroidism B.) PKU C.) Hypothyroidism D.) Thyroid storm

B

An unmarried young woman describes her sex life as active and involving many partners. She wants a contraceptive method that is reliable and does not interfere with sex. She requests an intrauterine device (IUD). Which information is most important for the nurse to share? A.) The IUD does not interfere with sex B.) The risk of pelvic inflammatory disease will be higher with the IUD C.) The IUD will protect you from sexually transmitted infections D.) Pregnancy rates are high with the IUD

B

Which clinical finding is a major use of ultrasonography in the first trimester? A.) Amniotic fluid volume B.) Presence of maternal abnormalities C.) Placental location and maturity D.) Cervical length

B

Which contraceptive method best protects against STIs and the HIV? A.) Periodic abstinence B.) Barrier methods C.) Hormonal methods D.) Same protection with all methods

B

Which statement by the student nurse regarding spermicides indicates effective learning? A.) "They are inserted 3 hours before sexual intercourse" B.) "They would be reapplied for each additional act of intercourse" C.) "Spermicides are added to male condoms to prevent pregnancy" D.) "Spermicides prevent sexually transmitted infections"

B

Which term describes the pregnant client who has a child and is in the 25th week of pregnancy? A.) Primipara B.) Multipara C.) Primigravida D.) Multigravida

B

Which treatment regime would be most appropriate for a client who has been recently diagnosed with acute pelvic inflammatory disease (PID)? A.) Oral antiviral therapy B.) Bed rest in a semi-Fowler position C.) Antibiotic regimen continued until symptoms subside D.) Frequent pelvic examination to monitor the healing progress

B

A client has arrived for her first prenatal appointment. She asked the nurse to explain exactly how long the pregnancy will be. What is the nurse's best response? A.) Normal pregnancy is 10 lunar months B.) Pregnancy is made up of four trimesters C.) Pregnancy is considered term at 36 weeks D.) Estimated date of delivery (EDD) is 40 completed weeks

A

For which condition is the client at risk in early pregnancy due to poorly controlled hyperglycemia? A.) Miscarriage B.) Hydramnios C.) Preeclampsia D.) Ketoacidosis

A

Of these psychosocial factors, which has the least negative effect on the health of the mother and/or fetus? A.) Moderate coffee consumption B.) Moderate alcohol consumption C.) Cigarette smoke D.) Emotional distress

A

The nurse is reviewing lab values to determine Rh incompatibility between mother and fetus. Which specific lab result should the nurse assess? A.) Indirect Coombs test B.) Hemoglobin level C.) Human chorionic gonadotropin (hCG) lecel D.) Maternal serum alpha-fetoprotein (MSAFP)

A

What is the primary role of the doula during labor? A.) Helps the woman perform Lamaze breathing techniques and to provide support to the woman and her partner B.) Checks the fetal monitor tracing for effects of the labor process on the fetal heart rate C.) Takes the place of the father as a coach and support provider D.) Administers pain medications as needed by the woman

A

What is the priority nursing action when planning a diet for a pregnant client? A.) Review the client's current dietary intake B.) Teach the client about MyPlate C.) Caution the client to avoid large doses of vitamins, especially those that are fat-soluble D.) Instruct the client to limit the intake of fatty foods

A

When assessing a pregnant client with thalassemia, the nurse knows that which factor is related to this condition? A.) An insufficient amount of hemoglobin is produced to fill the red blood cells (RBCs) B.) The RBCs have a normal life span that are sickled in shape C.) A folate deficiency occurs D.) There are inadequate levels of vitamin B12

A

Which additional caloric intake information is appropriate to include in the teaching for a client who is 6 weeks postpartum and breastfeeding? A.) 330 B.) 340 C.) 400 D.) 452

A

Which dietary intake is appropriate for the pregnant client who is still playing tennis at 32 weeks of gestation? A.) Several glasses of fluid B.) Extra protein sources, such as peanut butter C.) Salty foods to replace lost sodium D.) Easily digested sources of carbohydrate

A

Which factor is appropriate to identify when providing client instructions about daily monitoring of fetal kick counts? A.) Obesity B.) Alcohol C.) Smoking D.) Antidepressants

A

Which initial question is appropriate before explaining the use of cycle beads to a client using the Standard Day Method (SDM) of family planning to avoid pregnancy? A.) How often do you have menstrual cycles? B.) How do you currently track your menstrual cycle? C.) How does your partner feel about abstaining from intercourse during the fertile period? D.) Do you understand that this method has a higher failure rate than other methods of natural planning?

A

Which nursing information is appropriate to discuss with a client who is at 32 weeks of gestation? A.) Newborn care B.) Recommended exercise routine C.) Methods for pain management during labor D.) Maternal and fetal requirements

A

Which phase of the mother-child relationship is described as the client accepting the biologic fact of pregnancy and views the expectant child as part of herself? A.) Phase 1 B.) Phase 2 C.) Moratorium phase D.) Announcement phase

A

Which statement by the client indicates an understanding of the education regarding the concern of fetal exposure to high levels of mercury through maternal consumption of fish and seafood? A.) I will limit my intake of canned light tuna to 12 ounces per week B.) I will limit my intake of shark to no more than 6 ounces per week C.) I should only eat fish that has been caught by a commercial fisherman D.) I should avoid eating fish caught by my family if I am not sure the water is safe

A

Which type of cervical mucus would you expect in a non-fertile woman in postovulation? A.) Scant B.) Thick, cloudy and sticky C.) Clear, wet, sticky and slippery D.) Cloudy, yellow or white, and sticky

A

Which type of contraception is associated with toxic shock syndrome? A.) Diaphragm B.) Female condom C.) Intrauterine device D.) Contraceptive sponge

A

Which common complications are seen during the third trimester of pregnancy (select all that apply): A.) Constipation B.) Urinary frequency C.) Disturbance in sleep D.) Difficulty breathing E.) Nausea and vomiting

A, B, C, D

Which abnormal findings would the nurse immediately report to the health care provider for a client at 37 weeks of gestation who has been in a motor vehicle accident and is being discharged (select all that apply): A.) Leaking fluid B.) Abdominal pain C.) Vaginal bleeding D.) Irregular contractions E.) Decreased fetal movement

A, B, C, E

Which are bacterial sexually transmitted infections (STIs) (select all that apply): A.) Syphilis B.) Gonorrhea C.) Chlamydia tracheomatis D.) Vulvovaginal candidiasis E.) Pelvic inflammatory disease (PID)

A, B, C, E

Which nursing interventions are included in the plan of care for a pregnant client with mitral stenosis (select all that apply): A.) Restrict dietary sodium B.) Reduce activity C.) Assess the echocardiogram D.) Increase the client's activity E.) Assess the client's respiratory status

A, B, C, E

Which diet instruction about fish to avoid is appropriate for the pregnant client to prevent the risk of fetal neurotoxicity (select all that apply): A.) Shark B.) Tilefish C.) Catfish D.) Swordfish E.) Pollock

A, B, D

In assessing a pregnant client, the nurse is aware of the four high-risk factors when performing a health history. What are the four categories (select all that apply): A.) Biophysical B.) Psychosocial C.) Geographic D.) Enviornmental E.) Sociodemographic

A, B, D, E

The nurse is teaching the client insulin self-administration techniques. Which education would the nurse include in the lessons (select all that apply): A.) Allow the alcohol to dry before injecting the insulin B.) Puncture the skin at a 45-degree angle C.) Ensure that the insulin is injected rapidly D.) Apply gentle pressure after the injection E.) Record the dose and time of the injection

A, D, E

Which nursing information is appropriate to include when discussing dietary self-management for a client with hyperemesis (select all that apply): A.) Try to eat more dairy B.) Drink liquids from a cup with a straw C.) Eat a snack that is high with carbohydrates before bed D.) Consume protein after eating a sweet snack E.) Try drinking your water with a slice of lemon

A, D, E

A 39-year-old primigravida woman believes that she is approximately 8 weeks pregnant, although she has had irregular menstrual periods all her life. She has a history of smoking approximately one pack of cigarettes a day; however, she tells the nurse that she is trying to cut down. Her laboratory data are within normal limits. What diagnostic technique would be useful at this time? A.) Ultrasound examination B.) Maternal serum alpha-fetoprotein (MSAFP) screening C.) Amniocentesis D.) Nonstress test (NST)

A

Which contraceptive method is most often prescribed to clients who are on anticoagulant therapy for thromboembolism? A.) Estrogen-only pills B.) Progestin-only pills C.) Combined oral pills D.) Parenteral progestins

B

Which describes the nurse's understanding of the fetal risk for intrauterine growth restriction (IUGR)? A.) Asphyxia B.) Stillbirth C.) Traumatic birth injury D.) Asymmetric growth

B

Which risk is a fetus subject to if chorionic villus sampling (CVS) is conducted in the 7th week of gestation? A.) Reduced heart rate B.) Limb reduction defects C.) Decreased lung maturity D.) Neural tube defect

B

Which risk is associated with iron supplementation for the pregnant client? A.) Tetany B.) Anemia C.) Diabetes D.) Constipation

D

What is the correct term for a woman who has completed one pregnancy with a fetus (or fetuses) reaching the stage of fetal viability? A.) Primipara B.) Primigravida C.) Multipara D.) Nulligravida

A

Which behavior indicates that a woman is seeking safe passage for herself and her infant? A.) She keeps all prenatal appointments B.) She eats for two C.) She slowly drives her car D.) She wears only low-heeled shoes

A

Which condition in a client who is suing a copper-bearing intrauterine device indicates a need for removal of this device? A.) Rashes B.) Nausea C.) Occasional bleeding D.) Minor cramping

A

Which condition would require prophylaxis to prevent subacute bacterial endocarditis (SBE) both antepartum and intrapartum? A.) Valvular heart disease B.) Congestive heart disease C.) Arrhythmias D.) Postmyocardial infarction

A

Which initial assessment question is appropriate for a client who is trying to conceive a baby, is 3 days late for her period and has a negative pregnancy test? A.) Are you currently taking any medications B.) Did you take the test before you went to bed C.) Have you been able to conceive before this attempt D.) What type of birth control did you use before trying to conceive

A

Which interpretation is appropriate when assessing a client who is 7 months pregnant with increased chest movement and decreased abdominal movements while breathing? A.) Normal finding during pregnancy B.) Impaired diaphragm function C.) Decreased abdominal muscle tone D.) Presence of obstructive lung disease

A

Which is the most commonly used antenatal testing for a client at 39 weeks gestation who requires fetal assessment for decreased fetal movement? A.) Nonstress test B.) Biophysical profile C.) Contraction stress test D.) Vibroacoustic stimulation

A

Which is the most effective way to reduce the adverse consequences of sexually transmitted infections (STIs) for women and for society? A.) Preventing infection (primary prevention) B.) Preventing risky drug-related and sexual behaviors C.) Preventing STIs during pregnancy D.) Getting early treatment for any STIs

A

Which major neonatal complication is carefully monitored after the birth of the infant of a diabetic mother? A.) Hypoglycemia B.) Hypercalcemia C.) Hypobilirubinemia D.) Hypoinsulinemia

A

Which medication is recommended by the Centers for Disease Control and Prevention (CDC) for the treatment of chlamydia? A.) Doxycycline B.) Podofilox C.) Acyclovir D.) Penicillin

A

Which medication would be indicated for a client with human papillomavirus (HPV) infection? A.) Podofilox 0.5% solution B.) Azithromycin 1 g orally C.) Ceftriaxone 125 mg IM D.) Benzathine penicillin G 7.2 million units

A

Which nonpharmacologic contraceptive method has a failure rate of less than 25%? A.) Standard days variation B.) Periodic abstinence C.) Postovulation D.) Coitus interruptus

A

Which nursing response is appropriate for a heterosexual couple who asks if they need to use both a male and a female condom during sexual intercourse to reduce the risk for sexually transmitted infections? A.) "There is a possibility of both condoms tearing as a result of friction" B.) "You can use water-based lubricants with condoms" C.) "Using a female condom only may not provide protection" D.) "It will reduce the risk for sexually transmitted infections"

A

Which symptom is appropriate to assess for in the client who is taking an oral contraceptive and reports severe leg pain? A.) Thrombus formation B.) Severe muscle spasms C.) High creatinine levels D.) Hyperglycemic events

A

Which term best describes the conscious decision concerning when to conceive or avoid pregnancy as opposed to the intentional prevention of pregnancy during intercourse? A.) Family planning B.) Birth control C.) Contraception D.) Assisted reproductive therapy

A

With regard to medications, herbs, boosters, and other substances normally encountered by pregnant women, what is important for the nurse to be aware of? A.) Both prescription and over-the-counter (OTC) drugs that otherwise are harmless can be made hazardous by metabolic deficiencies of the fetus B.) The greatest danger of drug-caused developmental deficits in the fetus is observed in the final trimester C.) Killed-virus vaccines (e.g., tetanus) should not be administered during pregnancy, but live-virus vaccines (e.g., measles) are permissible D.) No convincing evidence exists that secondhand smoke is potentially dangerous to the fetus

A

A woman is in her seventh month of pregnancy. She has been complaining of nasal congestion and occasional epistaxis. Which statement best describes why this may be happening to this client? A.) This respiratory change is normal in pregnancy and caused by an elevated level of estrogen B.) This cardiovascular change is abnormal, and the nosebleeds are an ominous sign C.) The woman is a victim of domestic violence and is being hit in the face by her partner D.) The woman has been intranasally using cocaine

A

A woman is undergoing a nipple-stimulated CST. She is having contractions that occur every 3 minutes. The fetal heart rate (FHR) has a baseline heart rate of approximately 120 beats per minute without any decelerations. What is the correct interpretation of this test? A.) Negative B.) Positive C.) Satisfactory D.) Unsatisfactory

A

The nurse has formulated a diagnosis of Imbalanced nutrition: Less than body requirements for the client. Which goal is most appropriate for this client to obtain? A.) Gain a total of 30 pounds B.) Consistently take daily supplements C.) Decrease her intake of snack foods D.) Increase her intake of complex carbohydrates

A

The nurse is caring for a pregnant client who is on antibiotic therapy to treat a urinary tract infection (UTI). Which dietary changes would the nurse suggest for this client? A.) "Include yogurt, cheese and milk in your diet" B.) "Avoid folic acid supplements until the end of therapy" C.) "Include vitamins C and E supplementation in your diet" D.) "Reduce your dietary intake by 40 and 50 grams per day"

A

The nurse is caring for a pregnant client who is scheduled for surgery. Which nursing intervention would help provide sufficient fetal oxygenation during the surgery? A.) Positioning the client with a lateral tilt B.) Providing clear liquids C.) Palpating uterine contractions manually D.) Giving an antacid before administering anesthesia

A

Which hepatic changes are considered normal during pregnant (select all that apply): A.) Increased serum albumin B.) Increased serum alkaline phosphate C.) Increased serum cholesterol D.) Increased blood urea nitrogen E.) Increased nonprotein nitrogen

A, B, C

Which nursing instructions is included when educating female clients on how to use a contraceptive diaphragm (select all that apply): A.) "The diaphragm should completely cover your cervix" B.) "Empty your bladder before inserting the diaphragm' C.) "Remove the diaphragm 6 to 8 hours after intercourse" D.) "Insert the diaphragm up to 9 hours before sexual intercourse" E.) "Avoid the use of spermicide with the diaphragm"

A, B, C

Which sexual behaviors are associated with exposure to an STI (select all that apply): A.) Fellatio B.) Unprotected anal intercourse C.) Multiple sex partners D.) Dry kissing E.) Abstinence

A, B, C

Which signs and symptoms should a woman immediately report to her health care provider (select all that apply): A.) Vaginal bleeding B.) Rupture of membranes C.) Heartburn accompanied by severe headache D.) Decreased libido E.) Urinary frequency

A, B, C

Which statements are accurate regarding the most common bacterial sexually transmitted infections (select all that apply): A.) Chlamydial infections and gonorrhea are more likely to occur in women younger than 20 years of age B.) Gonorrhea can be transmitted to the newborn by direct contact with gonococcal organisms in the cervix C.) Syphilis can be transmitted through kissing, biting or oral-genital sex D.) Medications for pelvic inflammatory disease can be discontinued once symptoms disappear E.) Herpes is a common bacterial STI

A, B, C

Which are symptoms of a urinary tract infection (UTI) in the pregnant client (select all that apply): A.) Dysuria B.) Dribbling C.) Hematuria D.) Urinary frequency E.) Odor of vaginal discharge

A, B, C, D

Which assessments are included in the fetal BPP (select all that apply): A.) Fetal movement B.) Fetal tone C.) Fetal heart rate D.) AFI E.) Placental grade

A, B, C, D

Which nursing interventions are appropriate for a client who is learning to use a vaginal ring for the first time (select all that apply): A.) Encouraging the client to ask questions B.) Providing written instructions about the vaginal ring C.) Asking the client to perform a return demonstration D.) Providing information about emergency contraception E.) Asking the client to apply spermicidal foam to the ring before insertion

A, B, C, D

IUGR is associated with which pregnancy-related risk factors (select all that apply): A.) Poor nutrition B.) Maternal collagen disease C.) Gestational hypertension D.) Premature rupture of membranes E.) Smoking

A, B, C, E

Pregnancy is a hypercoagulable state in which women are at a fivefold to sixfold increased risk for thromboembolic disease. The tendency for blood to clot is greater, attributable to an increase in various clotting factors. Which of these come into play during pregnancy (select all that apply): A.) Factor VII B.) Factor VIII C.) Factor IX D.) Factor XIII E.) Fibrinogen

A, B, C, E

Which examples of protein-containing foods are appropriate when developing a dietary teaching plan for a client on a vegetarian diet (select all that apply): A.) Dried beans B.) Seeds C.) Peanut butter D.) Bagel E.) Peas

A, B, C, E

Which interventions would the nurse implement when providing care for a pregnant client with cystic fibrosis (select all that apply): A.) Assess the client's weight frequently B.) Assess for pulmonary infection C.) Assess for vitamin ADEK deficiency D.) Encourage exposure to sunlight E.) Monitor the fetal movements

A, B, C, E

The nurse is responsible for providing health teaching regarding the side effects of COCs. These side effects are attributed to estrogen, progesterone, or both. Which side effects are related to the use of COCs (select all that apply): A.) Gallbladder disease B.) Myocardial infarction and stroke C.) Hypotension D.) Breast tenderness and fluid retention E.) Dry skin and scalp

A, B, D

Diabetes refers to a group of metabolic diseases characterized by hyperglycemia resulting from defects in insulin action, insulin secretion, or both. Over time, diabetes causes significant changes in the microvascular and macrovascular circulations. What do these complications include (select all that apply): A.) Atherosclerosis B.) Retinopathy C.) Intrauterine fetal death (IUFD) D.) Nephropathy E.) Neuropathy F.) Autonomic neuropathy

A, B, D, E

Which factors contribute to the increased nutritional need during pregnancy (select all that apply): A.) Maternal mammary development B.) Increased maternal blood volume C.) Increased need for maternal fat stores D.) Increased metabolic rate E.) Development and growth of the uterine-placental-fetal unit

A, B, D, E

Which presumptive signs (felt by the woman) or probable sign (observed by the examiner) of pregnancy is not matched with another possible cause (select all that apply): A.) Amenorrhea: stress, endocrine problems B.) Quickening: gas, peristalsis C.) Goodell sign: cervical polyps D.) Chadwick sign: pelvic congestion E.) Urinary frequency: infection

A, B, D, E

A group of infections known collaboratively as TORCH infections are capable of crossing the placenta and causing serious prenatal effects on the fetus. Which infections are included in this group of organisms (select all that apply): A.) Toxoplasmosis B.) Other infections C.) Roseola D.) Clostridium E.) Herpes simplex

A, B, E

A woman has just moved to the United States from Mexico. She is 3 months pregnant and has arrived for her first prenatal visit. During her assessment interview, the nurse learns that the client has not had any immunizations. Which immunizations should she receive at this point in her pregnancy (select all that apply): A.) Tetanus B.) Diphtheria C.) Chickenpox D.) Rubella E.) Hepatitis B

A, B, E

Which are characteristics of Goodell sign (select all that apply): A.) Hypertrophy B.) Hyperplasia C.) Decreased friability D.) Decreased vascularity E.) Softening of the cervical tip

A, B, E

Which nursing action would be included in the initial treatment of a thyroid storm (select all that apply): A.) Oxygen B.) Intravenous fluids C.) Administration of iodide D.) Administration of dexamethasone E.) High doses of propylthiouracil (PTU)

A, B, E

Which nursing actions are included in the primary survey for a client at 19 weeks of gestation who has experienced a stab wound to the upper right abdominal quadrant and is not breathing (select all that apply): A.) Initiating an intravenous line B.) Administering oxygen C.) Performing an ultrasound examination D.) Evaluating the fetal heart rate E.) Using a jaw thrust to establish an airway

A, B, E

Which related signs are appropriate for the pregnant client whose hormonal reports reveal increased estrogen levels (select all that apply): A.) Mucoid discharge from the cervix B.) Heaviness in the breasts C.) Milk discharge from the nipples D.) Decreased chest expansion E.) Well-defined pink blotches on the palm

A, B, E

Which risks would the nurse expect to find in a pregnant client with a large ventricular septal defect (select all that apply): A.) Arrhythmias B.) Heart failure C.) Aortic dissection D.) Ineffective endocarditis E.) Pulmonary hypertension

A, B, E

The diagnosis of pregnancy is based on which positive signs of pregnancy (select all that apply): A.) Identification of fetal heartbeat B.) Palpation of fetal outline C.) Visualization of the fetus D.) Verification of fetal movement E.) Positive hCG test

A, C, D

When assessing a client, the nurse is aware of which manifestations associated with hypoglycemia (select all that apply): A.) Dizziness B.) Fruity breath C.) Blurred vision D.) Excessive hunger E.) Presence of acetone in urine

A, C, D

Which common micronutrient deficiencies are associated with bariatric surgery for the pregnant client (select all that apply): A.) Iron B.) Zinc C.) Folate D.) Calcium E.) Magnesium

A, C, D

Which statement about multifetal pregnancy is accurate (select all that apply): A.) The expectant mother often develops anemia because the fetuses have a greater demand for iron B.) Twin pregnancies come to term with the same frequency as single pregnancies C.) The mother should be counseled to increase her nutritional intake and gain more weight D.) Backache and varicose veins are more pronounced E.) There is always a history of fertility drugs

A, C, D

A lupus flare-up during pregnancy or early postpartum occurs in 15% to 60% of women with this disorder. Which conditions associated with systemic lupus erythematosus (SLE) are maternal risks (select all that apply): A.) Miscarriage B.) Intrauterine growth restriction (IUGR) C.) Nephritis D.) Preeclampsia E.) Cesarean birth

A, C, D, E

A pregnant client is being treated with penicillin G for syphilis. What client condition would the nurse immediately report to the primary care provider (select all that apply): A.) Fever B.) Nausea C.) Myalgias D.) Headache E.) Arthralgias

A, C, D, E

The nurse is aware of which subjective symptoms of cardiac decompensation when assessing a pregnant client (select all that apply): A.) Palpitations B.) Cyanosis of the lips C.) Rapid respiration D.) Difficulty breathing E.) Feeling of smothering

A, D, E

Which are prenatal tests used for diagnosing fetal defects in pregnancy (select all that apply): A.) Amniocentesis B.) Polyhydramnios C.) Amniotic fluid index (AFI) D.) Chorionic villus sampling (CVS) E.) Daily fetal movement count (DFMC) F.) Percutaneous umbilical blood sampling (PUBS)

A, D, F

A 25-year-old single woman comes to the gynecologists office for a follow-up visit related to her abnormal Papanicolaou (Pap) smear. The test revealed that the client has the HPV. The woman asks, What is that? Can you get rid of it? Which is the best response for the nurse to provide? A.) It's just a little lump on your cervix. We can just freeze it off B.) HPV stands for human papillomavirus. It is a sexually transmitted infection that may lead to cervical cancer C.) HPV is a type of early human immunodeficiency virus. You will die from this D.) You probably caught this from your current boyfriend. He should get tested for this

B

A 26-year-old primigravida has come to the clinic for her regular prenatal visit at 12 weeks. She appears thin and somewhat nervous. She reports that she eats a well-balanced diet, although her weight is 5 pounds less than it was at her last visit. The results of laboratory studies confirm that she has a hyperthyroid condition. Based on the available data, the nurse formulates a plan of care. Which nursing diagnosis is most appropriate for the client at this time? A.) Deficient fluid volume B.) Imbalanced nutrition: less than body requirements C.) Imbalanced nutrition: more than body requirements D.) Disturbed sleep pattern

B

A 41-week pregnant multigravida arrives at the labor and delivery unit after a NST indicated that her fetus could be experiencing some difficulties in utero. Which diagnostic tool yields more detailed information about the condition of the fetus? A.) Ultrasound for fetal anomalies B.) Biophysical profile (BPP) C.) MSAFP screening D.) Percutaneous umbilical blood sampling (PUBS)

B

A client is seen at the clinic at 14 weeks of gestation for a follow-up appointment. At which level does the nurse expect to palpate the fundus? A.) Nonpalpable above the symphysis at 14 weeks of gestation B.) Slightly above the symphysis pubis C.) At the level of the umbilicus D.) Slightly above the umbilicus

B

A woman who has a seizure disorder and takes barbiturates and phenytoin sodium daily asks the nurse about the pill as a contraceptive choice. What is the nurse's best response? A.) Oral contraceptives are a highly effective method, but they have some side effects B.) Your current medications will reduce the effectiveness of the pill C.) Oral contraceptives will reduce the effectiveness of your seizure medication D.) The pill is a good choice for a woman of your age and with your personal history

B

It is extremely rare for a woman to die in childbirth; however, it can happen. In the United States, the annual occurrence of maternal death is 12 per 100,000 cases of live birth. What are the leading causes of maternal death? A.) Embolism and preeclampsia B.) Trauma and motor vehicle accidents (MVAs) C.) Hemorrhage and infection D.) Underlying chronic conditions

B

Many clients are concerned about the increased levels of mercury in fish and may be reluctant to include this source of nutrients in their diet. What is the best advice for the nurse to provide? A.) Canned white tuna is a preferred choice B.) Shark, swordfish, and mackerel should be avoided C.) Fish caught in local waterways is the safest D.) Salmon and shrimp contain high levels of mercury

B

The musculoskeletal system adapts to the changes that occur throughout the pregnancy. Which musculoskeletal alteration should the client expect? A.) Her center of gravity will shift backward B.) She will have increased lordosis C.) She will have increased abdominal muscle tone D.) She will notice decreased mobility of her pelvic joints

B

The nurse is providing health education to a pregnant client regarding the cardiovascular system. Which information is correct and important to share? A.) A pregnant woman experiencing disturbed cardiac rhythm, such as sinus arrhythmia, requires close medical and obstetric observation no matter how healthy she may appear otherwise B.) Changes in heart size and position and increases in blood volume create auditory changes from 20 weeks of gestation to term C.) Palpitations are twice as likely to occur in twin gestations D.) All of the above changes will likely occur

B

The nurse knows which condition is associated with oligohydramnios? A.) Fetal hydrops B.) Potter syndrome C.) Neural tube defects D.) Fetal gastrointestinal obstruction

B

Which reason explains how a student nurse measures the blood pressure (BP) of a client as 170/90 mmHg, yet on reassessment the charge nurse finds that the BP is 110/70 mmHg? A.) Using a very large-sized cuff for measuring BP B.) Using a very small-sized cuff for measuring BP C.) Measuring the BP with the client in the lying position D.) Measuring the BP 30 minutes after tobacco consumption

B

Which reason is appropriate for a false positive result in the client with a history of epilepsy? A.) The test was taken too early B.) The client is taking tranquilizers C.) The client uses promethazine D.) The client is undergoing diuretic therapy

B

Which nursing instruction is appropriate when teaching a group of women about home pregnancy tests (select all that apply): A.) Use the last-void evening urine specimen B.) Follow the manufacturer's instructions carefully C.) Contact your primary health care provider for follow-up if the test result is negative D.) Repeat the test (if negative) after 1 week if amenorrhea persists E.) Contact your primary health care provider for follow-up if test result is positive

B, D, E

A client in the third trimester has just undergone an amniocentesis to determine fetal lung maturity. Which statement regarding this testing is important for the nurse in formulating a care plan? A.) Because of new imaging techniques, an amniocentesis should have been performed in the first trimester B.) Despite the use of ultrasonography, complications still occur in the mother or infant in 5% to 10% of cases C.) Administration of Rho(D) immunoglobulin may be necessary D.) The presence of meconium in the amniotic fluid is always a cause for concern

C

A woman arrives at the clinic for a pregnancy test. Her last menstrual period (LMP) was February 14, 2015. What is the clients expected date of birth (EDB)? A.) September 17, 2015 B.) November 7, 2015 C.) November 21, 2015 D.) December 17, 2015

C

A woman has experienced iron deficiency anemia during her pregnancy. She had been taking iron for 3 months before the birth. The client gave birth by cesarean 2 days earlier and has been having problems with constipation. After assisting her back to bed from the bathroom, the nurse notes that the woman's stools are dark (greenish-black). What should the nurse's initial action be? A.) Perform a guaiac test, and record the results B.) Recognize the finding as abnormal, and report it to the primary health care provider C.) Recognize the finding as a normal result of iron therapy C.) Check the woman's next stool to validate the observation

C

During the initial visit with a client who is beginning prenatal care, which action should be the highest priority for the nurse? A.) The first interview is a relaxed, get-acquainted affair during which the nurse gathers some general impressions of his or her new client B.) If the nurse observed handicapping conditions, he or she should be sensitive and not inquire about them because the client will do that in her own time C.) The nurse should be alert to the appearance of potential parenting problems, such as depression or lack of family support D.) Because of legal complications, the nurse should not ask about illegal drug use; that is left to the physician

C

If a client's normal prepregnancy diet contains 45 g of protein daily, how many more grams of protein should she consume per day during pregnancy? A.) 5 B.) 10 C.) 25 D.) 30

C

If consistently and correctly used, which of the barrier methods of contraception has the lowest failure rate? A.) Spermicides B.) Female condoms C.) Male condoms D.) Diaphragms

C

In the acronym BRAIDED, which letter is used to identify the key components of informed consent that the nurse must document? A.) B stands for birth control B.) R stands for reproduction C.) A stands for alternatives D.) I stands for ineffective

C

In the past, factors to determine whether a woman was likely to develop a high-risk pregnancy were primarily evaluated from a medical point of view. A broader, more comprehensive approach to high-risk pregnancy has been adopted today. Four categories have now been established, based on the threats to the health of the woman and the outcome of pregnancy. Which category should not be included in this group? A.) Biophysical B.) Psychosocial C.) Geographic D.) Environmental

C

The indirect Coombs test is a screening tool for Rh incompatibility. If the titer is greater than ______, amniocentesis may be a necessary next step. A.) 1:2 B.) 1:4 C.) 1:8 D.) 1:12

C

The labor and delivery nurse is preparing a client who is severely obese (bariatric) for an elective cesarean birth. Which piece of specialized equipment will not likely be needed when providing care for this pregnant woman? A.) Extra-long surgical instruments B.) Wide surgical table C.) Temporal thermometer D.) Increased diameter blood pressure cuff

C

The nurse is assessing a client at 30 weeks of pregnancy with poorly controlled pregestational diabetes. The client has come for the antenatal visit on a Monday. When would the nurse ask the client to come next? A.) The next week on a Monday B.) The next week on a Thursday C.) The same week on a Thursday D.) The week after next week on a Monday

C

The nurse observes that maternal hypotension has decreased uterine and fetal perfusion in a pregnant client. To further assess this situation, which factor would the nurse select to understand the maternal status? A.) D-dimer blood test B.) Kleihauer-Betke (KB) test C.) Electronic fetal monitoring D.) Electrocardiogram reading

C

The nurse should understand the process by which the HIV infection occurs. Once the virus has entered the body, what is the time frame for seroconversion to HIV positivity? A.) 6 to 10 days B.) 2 to 4 weeks C.) 6 to 12 weeks D.) 6 months

C

What is the drug of choice for the treatment of gonorrhea? A.) Penicillin G B.) Tetracycline C.) Ceftriaxone D.) Acyclovir

C

The nurse is teaching a client with gestational diabetes the technique to inject insulin. Which education would the nurse include (select all that apply): A.) Aspirate before injecting B.) Clean the injection site with alcohol C.) Insert the needle at a 90-degree angle D.) Inject the insulin slowly E.) After injection, cover the site with sterile gauze

C, D, E

A client states that she does not drink milk. Which foods should the nurse encourage this woman to consume in greater amounts to increase her calcium intake? A.) Fresh apricots B.) Canned clams C.) Spaghetti with meat sauce D.) Canned sardines

D

Nurses should be aware of the strengths and limitations of various biochemical assessments during pregnancy, including which items? A.) Chorionic villus sampling (CVS) is becoming more popular because it provides earlier diagnosis B.) Screening for maternal serum alpha-fetoprotein (MSAFP) levels is recommended only for women at risk for neural tube defects C.) Percutaneous umbilical blood sampling (PUBS) is one of the quad-screen tests for Down Syndrome D.) Maternal serum alpha-fetoprotein (MSAFP) is a screening tool only; it identified candidates for more definitive procedures

D

The nurse is caring for a pregnant client diagnosed with mitral valve stenosis. Which position would the nurse suggest to the client to ensure a safe labor? A.) Supine B.) Standing C.) Lithotomy D.) Side-lying

D

The nurse is teaching a client in the second trimester about fetal kick count. Which statement by the client needs correction? A.) "I should count the fetal kicks once a day for 60 minutes" B.) "I may not feel any movements when the fetus is sleeping" C.) "I should count the fetal kicks either after meals or before bedtime" D.) "I should consult an obstetrician if the fetal movements are less than 10 in one hour"

D

Which nursing information is recognized as part of the protocol of a perimortem cesarean delivery? A.) The goal is to deliver the neonate in less than 5 minutes B.) Maternal resuscitation cease after delivery of the neonate C.) The neonate must be delivered after 5 minutes of maternal resuscitation D.) After 4 minutes of maternal pulselessness, the neonate must be delivered

D

Which physiologic alteration of pregnancy most significantly affects glucose metabolism? A.) Pancreatic function in the islets of Langerhans is affected by pregnancy B.) Pregnant women use glucose at a more rapid rate than nonpregnant women C.) Pregnant women significantly increase their dietary intake D.) Placental hormones are antagonistic to insulin, thus resulting in insulin resistance

D

Which pregnant woman should strictly follow weight gain recommendations during pregnancy? A.) Pregnant with twins B.) In early adolescence C.) Shorter than 62 inches or 157 cm D.) Was 20 pounds overweight before pregnancy

D

Which type of cervical mucus would you expect when the woman is in preovulation? A.) Scant B.) Thick, cloudy and sticky C.) Clear, wet, sticky and slippery D.) Cloudy, yellow or white, and sticky

D

Which vitamins or minerals may lead to congenital malformations of the fetus if taken in excess by the mother? A.) Zinc B.) Vitamin D C.) Folic acid D.) Vitamin A

D

Why does the evaluation of abnormal Papanicolaou (Pap) tests during pregnancy become complicated? A.) The cervix is larger B.) The cervix is more oval in shape C.) The cervix is more soft and velvety D.) The squamocolumnar junction is located away from cervix

D

A 21-year-old client exhibits a greenish, copious, and malodorous discharge with vulvar irritation. A speculum examination and wet smear are performed. Which condition is this client most likely experiencing? A.) Bacterial vaginosis B.) Candidiasis C.) Yeast infection D.) Trichomoniasis

D

Which changes are considered the presumptive signs of pregnancy (select all that apply): A.) Fatigue B.) Hegar sign C.) Quickening D.) Amennorhea E.) Ballottment

A, C, D

To prevent gastrointestinal (GI) upset, when should a pregnant client be instructed to take the recommended iron supplements? A.) On a full stomach B.) At bedtime C.) After eating a meal D.) With milk

B

What is a subjective symptom of pregnancy? A.) Vaginal changes B.) Urinary frequency C.) Breast enlargement D.) Abdominal enlargement

B

Which nursing instructions are appropriate for a client who calls the clinic asking the nurse what to do for one missed combination oral contraceptive pill (select all that apply): A.) No backup contraceptive is needed B.) Take the next dose at the usual sign C.) Take one active pill as soon as possible D.) Take two pills and then resume one pill daily E.) Use a backup contraceptive for the next 7 days

A, B, C

Which are consequences of low estrogen levels in the pregnant client (select all that apply): A.) Peristalsis increases B.) Pepsin secretion decreases C.) Fat deposition is reduced D.) An epulis develops on the gum line E.) End arterioles branch

A, C

The nurse sees a primigravida at 30 weeks gestation for the first time. The nurse notes the client has smoked throughout the pregnancy, and fundal height measurements are suggestive of growth restriction in the fetus. In addition to ultrasound to measure fetal size, the nurse knows which tool is useful in confirming this condition? A.) Doppler blood flow analysis B.) Contraction stress test (CST) C.) Amniocentesis D.) Daily fetal movement counts

A

Which are known triggers for outbreaks for genital herpes (select all that apply): A.) Stress B.) Safety issues C.) Menstruation D.) Acute illness E.) Family illness

A, C

What diagnostic test would be used by the health care team to detect anencephaly in the fetus? A.) Amniocentesis B.) Doppler blood flow analysis C.) Nonstress test D.) Contraction stress test

A

Which dietary recommendations are appropriate to discuss with a pregnant client with acute viral hepatitis (select all that apply): A.) Low fat B.) Low sodium C.) High protein D.) Low cholesterol E.) High cholesterol

A, C

A pregnant client is being examined by the nurse in an outpatient obstetric clinic. The nurse suspects systemic lupus erythematosus (SLE) after the examination reveals which symptoms (select all that apply): A.) Muscle aches B.) Hyperactivity C.) Weight change D.) Fever E.) Hypotension

A, C, D

A 27-year-old pregnant woman had a preconceptual body mass index (BMI) of 19. What is this clients total recommended weight gain during pregnancy? A.) 20 kg (44 lb) B.) 16 kg (35 lb) C.) 12.5 kg (27.5 lb) D.) 10 kg (22 lb)

C

What type of cultural concern is the most likely deterrent to many women seeking prenatal care? A.) Religion B.) Modesty C.) Ignorance D.) Belief that physicians are evil

D

Who is most likely to experience the phenomenon of someone other than the mother-to-be having pregnancy-like symptoms such as nausea and weight gain? A.) Mother of the pregnant woman B.) Couples teenage daughter C.) Sister of the pregnant woman D.) Expectant father

D

A first-time mother at 18 weeks of gestation is in for her regularly scheduled prenatal visit. The client tells the nurse that she is afraid that she is going into premature labor because she is beginning to have regular contractions. The nurse explains that these are Braxton Hicks contractions. What other information is important for the nurse to share? A.) Braxton Hicks contractions should be painless B.) They may increase in frequency with walking C.) These contractions might cause cervical dilation D.) Braxton Hicks contractions will impede oxygen flow to the fetus

A

A nurse is caring for a client with mitral stenosis who is in the active stage of labor. Which action would the nurse take to promote cardiac function? A.) Maintain the client in a side-lying position with the head and shoulders elevated to facilitate hemodynamics B.) Prepare the client for cesarean delivery because this is the recommended method to sustain hemodynamics C.) Encourage the client to avoid the use of narcotics or epidural regional analgesia because this alters cardiac function D.) Promote the use of the valsalva maneuver during pushing in the second stage to improve diastolic ventricular filling

A

A pregnant client reports abdominal pain in the right lower quadrant, along with nausea and vomiting. The urinalysis report shows an absence of urinary tract infection. A chest x-ray also rules out lower-lobe pneumonia. Which client condition would the nurse suspect? A.) Appendicitis B.) Cholethiasis C.) Placenta previa D.) Uterine rupture

A

A pregnant client tells her nurse that she is worried about the blotchy, brownish coloring over her cheeks, nose, and forehead. The nurse can reassure her that this is a normal condition related to hormonal changes. What is the correct term for this integumentary finding? A.) Melasma B.) Linea nigra C.) Striae gravidarum D.) Palmar erythema

A

A woman has chosen the calendar method of conception control. Which is the most important action the nurse should perform during the assessment process? A.) Obtain a history of the woman's menstrual cycle lengths for the past 6 to 12 months B.) Determine the client's weight gain and loss pattern for the previous year C.) Examine skin pigmentation and hair texture for hormonal changes D.) Explore the client' s previous experiences with conception control

A

After a woman with blood type Rh negative undergoes amniocentesis, the most appropriate nursing intervention is to: A.) Administer RhoD immunoglobulin B.) Administer anticoagulant C.) Send the patient for a computed tomography (CT) scan before the procedure D.) Assure the mother that short-term radiation exposure is not harmful to the fetus

A

After reviewing a client's blood glucose levels, the nurse finds that the client is hypoglycemic and administers three glucose tablets. After 15 minutes, the client is still hypoglycemic, so the nurse administers three more glucose tablets. What would the nurse do next if the client's blood glucose is 60 mg/dL after 15 minutes? A.) Notify the primary health care provider B.) Administer a 50% dextrose intravenous push C.) Obtain blood samples for blood gas analysis D.) Give three more glucose tablets for the client

A

After reviewing the medical history and dietary habits of a pregnant client, the nurse suspects that the client's newborn may have risk of hyperactivity and learning disabilities due to which factors? A.) The client consumed alcohol during the pregnancy B.) The client has three pregnancies in two years C.) The client drinks 6 ounces of coffee daily D.) The client is on valproic acid therapy

A

After the nurse completes nutritional counseling for a pregnant woman, she asks the client to repeat the instructions to assess the clients understanding. Which statement indicates that the client understands the role of protein in her pregnancy? A.) Protein will help my baby grow B.) Eating protein will prevent me from becoming anemic C.) Eating protein will make my baby have strong teeth after he is born D.) Eating protein will prevent me from being diabetic

A

Although reported in small numbers, toxic shock syndrome (TSS) can occur with the use of a diaphragm. If a client is interested in this form of conception control, then the nurse must instruct the woman on how best to reduce her risk of TSS. Which comment by the nurse would be most helpful in achieving this goal? A.) You should always remove your diaphragm 6 to 8 hours after intercourse. Don't use the diaphragm during menses, and watch for danger signs of TSS, including a sudden onset of fever over 38.4 C, hypotension, and a rash B.) You should remove your diaphragm right after intercourse to prevent TSS C.) It's okay to use your diaphragm during your menstrual cycle. Just be sure to wash it thoroughly first to prevent TSS D.) Make sure you don't leave your diaphragm in for longer than 24 hours, or you may get TSS

A

An essential component of counseling women regarding safe sex practices includes a discussion regarding avoiding the exchange of body fluids. The most effective physical barrier promoted for the prevention of STIs and HIV is the condom. To educate the client about the use of condoms, which information related to condom use is the most important? A.) Strategies to enhance condom use B.) Choice of colors and special features C.) Leaving the decision up to the male partner D.) Places to carry condoms safely

A

Clients treated for syphilis with penicillin may experience a Jarisch-Herxheimer reaction. Which clinical presentation would be unlikely if a client is experiencing this reaction? A.) Vomiting and diarrhea B.) Headache, myalgias, and arthralgia C.) Preterm labor D.) Jarisch-Herxheimer in the first 24 hours after treatment

A

The health history and physical examination cannot reliably identify all persons infected with HIV or other blood-borne pathogens. Which infection control practice should the nurse use when providing eye prophylaxis to a term newborn? A.) Wear gloves B.) Wear mouth, nose, and eye protection C.) Wear a mask D.) Wash the hands after medication administration

A

The human papillomavirus (HPV), also known as genital warts, affects 79 million Americans, with an estimated number of 14 million new infections each year. The highest rate of infection occurs in young women, ages 20 to 24 years. Prophylactic vaccination to prevent the HPV is now available. Which statement regarding this vaccine is inaccurate? A.) Only one vaccine for the HPV is available B.) The vaccine is given in three doses over a 6-month period C.) The vaccine is recommended for both boys and girls D.) Ideally, the vaccine is administered before the first sexual contact

A

The nurse is caring for a pregnant client with gestational diabetes. What would the nurse teach the client about diet during pregnancy? A.) Eat three meals a day with two or three snacks B.) Avoid meals or snacks just before bedtime C.) Use artificial sweeteners instead of sugar D.) Avoid foods that are high in dietary fiber

A

The nurse is screening a pregnant client and is aware that which factor is known to increase the risk of gestational diabetes mellitus? A.) Previous birth of large infant B.) Maternal age younger than 25 years C.) Underweight before pregnancy D.) Previous diagnosis of type 2 diabetes mellitus

A

The nurse is supervising a student nurse while performing a nonstress test. Which action of the student nurse indicates the need for further teaching? A.) Placing the client in the left side-lying position B.) Instructing the client to press the handheld marker C.) Offering glucose water prior to the test D.) Applying a tocodynamometer with Doppler transducer

A

The nurse is teaching a pregnant client regarding the monitoring of daily fetal movement. Which finding should be reported to the primary health care provider? A.) Fetal movement was not declared for 12 hours B.) An episode of limb straightening was observed C.) One episode of fetal breathing was seen in 30 minutes D.) An amniotic fluid index value of more than 5 cm

A

The nurse is teaching a student nurse about maternal cardiac risk groups and mortality rates. Which statement is accurate regarding risk groups and mortality rates? A.) Aortic stenosis has a mortality rate of 5% to 15% B.) Pulmonary hypertension is categorized at group I C.) Patent ductus arteriosus is categorized as group II D.) Artificial heart valves has a mortality rate of less than 1%

A

The nurse sees a woman for the first time when she is 30 weeks pregnant. The client has smoked throughout the pregnancy, and fundal height measurements now are suggestive of intrauterine growth restriction (IUGR) in the fetus. In addition to ultrasound to measure fetal size, what is another tool useful in confirming the diagnosis? A.) Doppler blood flow analysis B.) Contraction stress test (CST) C.) Amniocentesis D.) Daily fetal movement counts

A

What is important for the nurse to recognize regarding the new father and his acceptance of the pregnancy and preparation for childbirth? A.) The father goes through three phases of acceptance of his own B.) The fathers attachment to the fetus cannot be as strong as that of the mother because it does not start until after the birth C.) In the last 2 months of pregnancy, most expectant fathers suddenly get very protective of their established lifestyle and resist making changes to the home D.) Typically, men remain ambivalent about fatherhood right up to the birth of their child

A

Which analysis of maternal serum may predict chromosomal abnormalities in the fetus? A.) Multiple-marker screening B.) L/S ratio C.) BPP D.) Blood type and crossmatch of maternal and fetal serum

A

Which condition is likely to be identified by the quadruple marker screen? A.) Down syndrome B.) Diaphragmatic hernia C.) Congenital cardiac abnormality D.) Anencephaly

A

Which describes the use for ultrasonography in the third trimester of pregnancy (select all that apply): A.) Detects congenital anomalies B.) Assess the placental location C.) Detects maternal uterine abnormalities D.) Determines the causes of vaginal bleeding E.) Determines the gestational age of the fetus

A

Which guidance might the nurse provide for a client with severe morning sickness? A.) Trying lemonade and potato chips B.) Drinking plenty of fluids early in the day C.) Immediately brushing her teeth after eating D.) Never snacking before bedtime

A

Which information is appropriate for the nurse to be aware of with regard to medications, herbs, shots and other substances normally encountered? A.) Prescription and over-the-counter (OTC) drugs that otherwise are harmless can be hazardous because they can cross the placenta and pose a risk to the developing fetus B.) The greatest danger of drug-caused developmental deficits in the fetus is seen in the final trimester C.) Killed-virus vaccines (e.g. tetanus) should not be given during pregnancy, but live-virus vaccines (measles) are permissible D.) No convincing evidences exists that secondhand smoke is potentially dangerous to the fetus

A

Which information is the highest priority for the nurse to comprehend regarding the BPP? A.) BPP is an accurate indicator of impending fetal well-being B.) BPP is a compilation of health risk factors of the mother during the later stages of pregnancy C.) BPP consists of a Doppler blood flow analysis and an amniotic fluid index (AFI) D.) BPP involves an invasive form of an ultrasonic examination

A

Which information regarding protein in the diet of a pregnant woman is most helpful to the client? A.) Many protein-rich foods are also good sources of calcium, iron, and B vitamins B.) Many women need to increase their protein intake during pregnancy C.) As with carbohydrates and fat, no specific recommendations exist for the amount of protein in the diet D.) High-protein supplements can be used without risk by women on macrobiotic diets

A

Which instruction is appropriate for a client who takes combined oral contraceptives (COCs)? A.) "Take only 1 pill at the same time each day" B.) "You may have increased menstrual blood flow" C.) "Iron-deficiency anemia is a side effect of the pill" D.) "The pill increases the risk for endometrial cancer"

A

Which nursing action is appropriate to prevent conception for the client who has regular menstrual cycles every 28 days? A.) Abstain from sexual intercourse from days 10 to 17 B.) Abstain from sexual intercourse from days 6 to 19 C.) It is safe to have unprotected sexual intercourse from days 11 to 17 D.) It is safe to have unprotected sexual intercourse from days 12 to 16

A

Which nursing advice is appropriate for the client using the ParaGuard Copper T 380A? A.) "Report immediately if rashes occur" B.) "Drink warm water fi you experience cramping" C.) "Irregular spotting is a serious side effect and needs prompt treatment" D.) "Use a condom to increase contraceptive effectiveness"

A

Which nursing information is appropriate regarding a father's acceptance of the pregnancy and preparation for childbirth? A.) The father goes through three phases of acceptance of his own B.) The father's attachment to the fetus cannot be as strong as that of the mother because it does not start until after birth C.) In the last 2 months of pregnancy, most expectant fathers suddenly get very protective of their established lifestyle and resist making changes to the home D.) Typically men remain ambivalent about fatherhood right up to the birth of their child

A

Which nursing information is appropriate regarding protein in the diet of pregnant clients? A.) Many protein-rich foods are good sources of calcium, iron and B vitamins B.) Many women need to increase their protein intake during pregnancy C.) As with carbohydrates and fat, no specific recommendations exists for the amount of protein in the diet D.) High-protein supplements can be used without risk on a macrobiotic diet

A

Which nutrients recommended dietary allowance (RDA) is higher during lactation than during pregnancy? A.) Energy (kcal) B.) Iron C.) Vitamin A D.) Folic acid

A

Which nutritional recommendation regarding fluids is accurate? A.) A woman's daily intake should be six to eight glasses of water, milk, and/or juice B.) Coffee should be limited to no more than 2 cups, but tea and cocoa can be consumed without worry C.) Of the artificial sweeteners, only aspartame has not been associated with any maternity health concerns D.) Water with fluoride is especially encouraged because it reduces the child's risk of tooth decay

A

Which presumptive sign or symptom of pregnancy would a client experience who is approximately 10 weeks of gestation? A.) Amenorrhea B.) Positive pregnancy test C.) Chadwick sign D.) Hegar sign

A

Which statement by the client indicates the need for further teaching about the use of condoms? A.) "Condoms can prevent me from getting STIs" B.) "I can use lubricant-based jelly if I need to" C.) "I will check the condom for tears after intercourse" D.) "Condoms may reduce sensation during intercourse"

A

Which statement concerning the complication of maternal diabetes is the most accurate? A.) Diabetic ketoacidosis (DKA) can lead to fetal death at any time during pregnancy B.) Hydramnios occurs approximately twice as often in diabetic pregnancies than in nondiabetic pregnancies C.) Infections occur about as often and are considered about as serious in both diabetic and nondiabetic pregnancies D.) Even mild-to-moderate hypoglycemic episodes can have significant effects on fetal well-being

A

Which statement regarding emergency contraception is correct? A.) Emergency contraception requires that the first dose be taken within 72 hours of unprotected intercourse B.) Emergency contraception may be taken right after ovulation C.) Emergency contraception has an effectiveness rate in preventing pregnancy of approximately 50% D.) Emergency contraception is commonly associated with the side effect of menorrhagia

A

Which type of ultrasound is the standard medical scan used in pregnancy? A.) Two-dimensional (2D) B.) Three-dimensional (3D) C.) Four-dimensional (4D) D.) Five-dimensional (5D)

A

Which viral sexually transmitted infection is characterized by a primary infection followed by recurrent episodes? A.) Herpes simplex virus 2 (HSV-2) B.) HPV C.) HIV D.) CMV

A

Which women should undergo prenatal testing for the human immunodeficiency virus (HIV)? A.) All women, regardless of risk factors B.) Women who have had more than one sexual partner C.) Women who have had a sexually transmitted infection (STI) D.) Woman who are monogamous with one partner

A

Why might it be more difficult to diagnose appendicitis during pregnancy? A.) The appendix is displaced upward and laterally, high and to the right B.) The appendix is displaced upward and laterally, high and to the left C.) The appendix is deep at the McBurneys point D.) The appendix is displaced downward and laterally, low and to the right

A

The nurse is advising a pregnant client who has been prescribed lispro. Which information would the nurse provide about the insulin (select all that apply): A.) It is rapid-acting insulin preferred for use during pregnancy B.) It is injected just before meals and causes less hyperglycemia C.) It has shorter duration of action as compared to regular insulin D.) It is released slowly in small amounts with no pronounced peak E.) Its action lasts for 12 hours maintaining optimal blood glucose levels

A, B, C

Most women with uncomplicated pregnancies can use the nurse as their primary source for nutritional information. However, the nurse or midwife may need to refer a client to a registered dietitian for in-depth nutritional counseling. Which conditions would require such a consultation (select all that apply): A.) Preexisting or gestational illness such as diabetes B.) Ethnic or cultural food patterns C.) Obesity D.) Vegetarian diets E.) Multifetal pregnancy

A, B, C, D

A 23-year-old primiparous client with inconsistent prenatal care is admitted to the hospitals maternity unit in labor. The client states that she has tested positive for the HIV. She has not undergone any treatment during her pregnancy. The nurse understands that the risk of perinatal transmission can be significantly decreased by a number of prophylactic interventions. Which interventions should be included in the plan of care (select all that apply): A.) Intrapartum treatment with antiviral medications B.) Cesarean birth C.) Postpartum treatment with antiviral medications D.) Avoidance of breastfeeding E.) Pneumococcal, HBV, and Haemophilus influenzae vaccine

A, B, D

Which nursing interventions are appropriate to relieve symptoms of nausea for the pregnant client in her first trimester of pregnancy (select all that apply): A.) Assessing if the client is well hydrated B.) Assessing the client's weight gain pattern during pregnancy C.) Reviewing the food frequency approach during pregnancy D.) Reviewing measures already taken for the prevention of morning sickness E.) Discussing food cravings that may occur during pregnancy

A, B, D

Which actions are low-risk sexual practices (select all that apply): A.) Abstinence B.) Dry kissing C.) Wet kissing D.) Mutual masturbation E.) Hugging, massaging, touching (assuming no break in skin) F.) Monogamous (both partners and no high-risk activities) but not tested for human immunodeficiency virus (HIV) or other sexually transmitted in sexually transmitted infections

A, B, D, E

A woman is in for a routine prenatal checkup. The nurse is assessing her urine for glycosuria and proteinuria. Which findings are considered normal (select all that apply): A.) Dipstick assessment of trace to +1 B.) <300 mg/24 hours C.) Dipstick assessment of +2 glucose D.) >300 mg/24 hours E.) Albumin < 30 mg/24 hours

A, B, E

A serious but uncommon complication of undiagnosed or partially treated hyperthyroidism is a thyroid storm, which may occur in response to stress such as infection, birth, or surgery. What are the signs and symptoms of this emergency disorder (select all that apply): A.) Fever B.) Hypothermia C.) Restlessness D.) Bradycardia E.) Hypertension

A, C

In caring for a pregnant woman with sickle cell anemia, the nurse must be aware of the signs and symptoms of a sickle cell crisis. What do these include (select all that apply): A.) Fever B.) Endometritis C.) Abdominal pain D.) Joint pain E.) Urinary tract infection (UTI)

A, C, D

The nurse should be familiar with the use of the five Ps as a tool for evaluating risk behaviors for STIs and the HIV. Which components would the nurse include in her use of the five Ps as an assessment tool (select all that apply): A.) Number of partners B.) Level of physical activity C.) Prevention of pregnancy D.) Protection from STIs E.) Past history

A, C, D, E

Which symptoms are associated with syphilis (select all that apply): A.) Generalized lymphadenopathy B.) Abdominal pain and irregular bleeding C.) Warlike infectious lesions on the vulva D.) A painful papule at the site of inoculation E.) The presence of rash on the palms and soles

A, C, E

The nurse is caring for a postpartum breastfeeding client with asthma who has been prescribed theophylline. Which complications would the nurse assess for in the newborn (select all that apply): A.) Jitteriness B.) Hyponatremia C.) Cooley's anemia D.) Cardiac arrhythmias E.) Vitamin A deficiency

A, D

The nurse is reviewing the educational packet provided to a client about tubal ligation. Which information regarding this procedure is important for the nurse to share (select all that apply): A.) It is highly unlikely that you will become pregnant after the procedure B.) Tubal ligation is an effective form of 100% permanent sterilization. You won't be able to get pregnant C.) Sterilization offers some form of protection against STIs D.) Sterilization offers no protection against STIs E.) Your menstrual cycle will greatly increase after your sterilization

A, D

Which vaccinations reduce the risk for contracting sexually transmitted infections (STIs) (select all that apply): A.) Hepatitis B B.) Pneumococcal conjugate C.) Meningococcal conjugate D.) Human papillomavirus (HPV) E.) Pneumococcal polysaccharide

A, D

Which symptoms of hyperglycemia are appropriate to include in the teaching for a diabetic client (select all that apply): A.) Thirst B.) Hunger C.) Fatigue D.) Drowsiness E.) Constipation

A, D, E

Which are some safety measures to take while pregnant (select all that apply): A.) Use correct body mechanics B.) Avoid travel to high-altitude regions about 1000 feet C.) Perform activities requiring coordination, balance and concentration D.) Take rest periods; reschedule daily activities to meet rest and relaxation needs E.) Avoid environmental teratogens, such as cleaning agents, paints, sprays, herbicides and pesticides F.) Use safety features on tools and vehicles (e.g. safety seat belts, shoulder harnesses, headrests, goggles, helmets) are specified

A, D, E, F

Place in order the developmental tasks required to achieve maternal adaptation: A.) Preparing for the birth experience B.) Establishing a relationship with the unborn child C.) Reordering the relationship between herself and her partner D.) Identifying with the role of mother E.) Accepting the pregnancy F.) Reordering a the relationship between herself and her mother

A, D, F, C, B, A

During the physical examination of a client beginning prenatal care, which initial action is most important for the nurse to perform? A.) Only women who show physical signs or meet the sociologic profile should be assessed for physical abuse B.) The client should empty her bladder before the pelvic examination C.) The distribution, amount, and quality of body hair are of no particular importance D.) The size of the uterus is discounted in the initial examination because it will be increasing in size during the second trimester

B

Five different viruses (A, B, C, D, and E) account for almost all cases of hepatitis infections. Which statement regarding the various forms of hepatitis is most accurate? A.) Vaccine exists for hepatitis C virus (HCV) but not for HBV B.) HAV is acquired by eating contaminated food or drinking polluted water C.) HBV is less contagious than HIV D.) Incidence of HCV is decreasing

B

Importantly, the nurse must be aware of which information related to the use of IUDs? A.) Return to fertility can take several weeks after the device is removed B.) IUDs containing copper can provide an emergency contraception option if inserted within a few days of unprotected intercourse C.) IUDs offer the same protection against STIs as the diaphragm D.) Consent forms are not needed for IUD insertion

B

Maternal nutritional status is an especially significant factor of the many that influence the outcome of pregnancy. Why is this the case? A.) Maternal nutritional status is extremely difficult to adjust because of an individuals ingrained eating habits B.) Adequate nutrition is an important preventive measure for a variety of problems C.) Women love obsessing about their weight and diets D.) A woman's preconception weight becomes irrelevant

B

Nurses, certified nurse-midwives, and other advanced practice nurses have the knowledge and expertise to assist women in making informed choices regarding contraception. A multidisciplinary approach should ensure that the woman's social, cultural, and interpersonal needs are met. Which action should the nurse first take when meeting with a new client to discuss contraception? A.) Obtain data about the frequency of coitus B.) Determine the woman's level of knowledge concerning contraception and her commitment to any particular method C.) Assess the woman's willingness to touch her genitals and cervical mucus D.) Evaluate the woman's contraceptive life plan

B

Of which physiologic alteration of the uterus during pregnancy is it important for the nurse to alert the patient? A.) Lightening occurs near the end of the second trimester as the uterus rises into a different position B.) Woman's increased urinary frequency in the first trimester is the result of exaggerated uterine anti-reflexion caused by softening C.) Braxton Hicks contractions become more painful in the third trimester, particularly if the woman tries to exercise D.) Uterine souffle is the movement of the fetus

B

On reviewing the medical history of a pregnant client, the nurse finds that the client is taking carbamazepine. What consequence of the drug on the fetus should the nurse be aware of? A.) Pylectasis B.) Spina bifida C.) Omphalocele D.) Lupus erythematosus

B

Preconception counseling is critical in the safe management of diabetic pregnancies. Which complication is commonly associated with poor glycemic control before and during early pregnancy? A.) Frequent episodes of maternal hypoglycemia B.) Congenital anomalies in the fetus C.) Hydramnios D.) Hyperemesis gravidarum

B

The Centers for Disease Control and Prevention (CDC) recommends which therapy for the treatment of the HPV? A.) Miconazole ointment B.) Topical podofilox 0.5% solution or gel C.) Two doses of penicillin administered intramuscularly (IM) D.) Metronidazole by mouth

B

The client is instructed to place her thumb and forefinger on the areola and gently press inward. What is the purpose of this exercise? A.) To check the sensitivity of the nipples B.) To determine whether the nipple is everted or inverted C.) To calculate the adipose buildup in the abdomen D.) To see whether the fetus has become inactive

B

The lactational amenorrhea method (LAM) of birth control is popular in developing countries and has had limited use in the United States. As breastfeeding rates increase, more women may rely upon this method for birth control. Which information is most important to provide to the client interested in using the LAM for contraception? A.) LAM is effective until the infant is 9 months of age B.) This popular method of birth control works best if the mother is exclusively breastfeeding C.) Its typical failure rate is 5% D.) Feeding intervals should be 6 hours during the day

B

The nurse is caring for a client in the first trimester of pregnancy. The client's laboratory reports indicate a reduction in the levels of pregnancy-associated placental protein (PAPP-A) and an elevation in the levels of human chorionic gonadotropin (hCG) and nuchal translucency (NT). Based on these findings, which condition does the nurse suspect in the fetus? A.) Spina bifida B.) Down syndrome C.) Potter syndrome D.) Fetal cardiac disease

B

The nurse is explaining a Doppler flow analysis screening to a pregnant client. In describing the test, the nurse states at which stage of the pregnancy can the systolic/diastolic ratio be first directed in the fetus? A.) 2nd week of pregnancy B.) 15th week of pregnancy C.) 24th week of pregnancy D.) 32nd week of pregnancy

B

The nurse is planning the care for a laboring client with diabetes mellitus. This client is at greater risk for which clinical finding? A.) Oligohydramnios B.) Polyhydramnios C.) Postterm pregnancy D.) Chromosomal abnormalities

B

The nurse is reviewing the amniocentesis reports of a client at 20 weeks gestation and notes the presence of high alpha-fetoprotein (AFP) levels. Which should the nurse infer from this information related to the clinical condition of the fetus? A.) Cardiac disorder B.) Neurologic disorder C.) Circulatory disorder D.) Pulmonary disorder

B

The nurse notices that a pregnant client shows signs of fatigue and lethargy, and has glossitis and rough skin. Which condition should the nurse likely suspect? A.) Thalassemia B.) Megaloblastic anemia C.) Iron deficiency anemia D.) Sickle cell hemoglobinopathy

B

The nurse who is caring for a woman hospitalized for hyperemesis gravidarum would expect the initial treatment to involve what? A.) Corticosteroids to reduce inflammation B.) Intravenous (IV) therapy to correct fluid and electrolyte imbalances C.) Antiemetic medication, such as pyridoxine, to control nausea and vomiting D.) Enteral nutrition to correct nutritional deficits

B

The nurse working with pregnant clients must seek to gain understanding of the process whereby women accept their pregnancy. Which statement regarding this process is most accurate? A.) Nonacceptance of the pregnancy very often equates to a rejection of the child B.) Mood swings are most likely the result of worries about finances and a changed lifestyle, as well as profound hormonal changes C.) Ambivalent feelings during pregnancy are usually only expressed in emotionally immature or very young mothers D.) Conflicts such as not wanting to be pregnant or childrearing and career-related decisions need not be addressed during pregnancy because they will naturally resolve themselves after birth

B

When assessing a pregnant client with heart disease, the nurse knows that which form of heart disease in women of childbearing years usually has a benign effect on pregnancy? A.) Cardiomyopathy B.) Mitral valve prolapse C.) Rheumatic heart disease D.) Congenital heart disease

B

When counseling a pregnant client with asthma, the nurse knows that the severity of symptoms usually peak during which time? A.) In the first trimester B.) Between 17 and 24 weeks of gestation C.) During the last 4 weeks of pregnancy D.) Immediately postpartum

B

When would the nurse expect a client to develop a viremic influenza-like response after being infected with human immunodeficiency virus (HIV)? A.) Immediately after the virus infects the body B.) Within 6 to 12 weeks after the virus infects the body C.) 1 year after initiating antiretroviral therapy D.) Immediately after initiating antiretroviral therapy

B

Which client is a probable candidate for the Essure system of transcervical sterilization? A.) A client who has a history of ectopic pregnancy B.) An obese client with abdominal adhesion C.) A client who had a vaginal delivery 1 week ago D.) A client who wants more children

B

Which client might be well advised to continue condom use during intercourse throughout her pregnancy? A.) Unmarried pregnant women B.) Women at risk for acquiring or transmitting STIs C.) All pregnant women D.) Women at risk for candidiasis

B

Which client statement indicates a need for further teaching about sibling adaptation? A.) Show the child how to touch the baby B.) Exclude the child during infant feeding times C.) Don't force interactions between the child and the baby D.) Help the child have realistic expectations about the baby

B

Which condition is a contraindication when using the spermicide nonoxynol-9 (N-9) as, or with, a client's contraceptive method? A.) Iron-deficiency anemia B.) Risk for human immunodeficiency virus (HIV) C.) Contraceptive diaphragm use D.) Condom use of the partner

B

Which condition is expected after a laboratory report for a pregnant client shows low levels of serum ferritin? A.) Tetany B.) Anemia C.) Renal failure D.) Hypertension

B

Which condition is related to inadequate weight gain during pregnancy? A.) Spina bifida B.) Intrauterine growth restriction C.) Diabetes mellitus D.) Down syndrome

B

Which education would the nurse provide clients about self-care prevention of genital tract infections? A.) Increase dietary sugar and avoid yogurt B.) Limit time spent in damp-exercise clothes and limit exposure to bath salts or bubble bath C.) Choose underwear or hosiery with a nylon crotch D.) Douche frequently

B

Which finding is considered normal for a client in her second trimester? A.) Less audible heart sounds (S1, S2) B.) Increased pulse rate C.) Increased blood pressure D.) Decreased red blood cell (RBC) production

B

Which food item is appropriate to include in the vitamin B6-deficient pregnant client's diet to encourage normal fetal development? A.) Yeast B.) Meat C.) Asparagus D.) Strawberries

B

Which information regarding the care of antepartum women with cardiac conditions is most important for the nurse to understand? A.) Stress on the heart is greatest in the first trimester and the last 2 weeks before labor B.) Women with class II cardiac disease should avoid heavy exertion and any activity that causes even minor symptoms C.) Women with class III cardiac disease should get 8 to 10 hours of sleep every day and limit housework, shopping, and exercise D.) Women with class I cardiac disease need bed rest through most of the pregnancy and face the possibility of hospitalization near term

B

Which information should the nurse take into consideration when planning care for a postpartum client with cardiac disease? A.) The plan of care for a postpartum client is the same as the plan for any pregnant woman B.) The plan of care includes rest, stool softeners, and monitoring of the effect of activity C.) The plan of care includes frequent ambulating, alternating with active range-of-motion exercises D.) The plan of care includes limiting visits with the infant to once per day

B

Which instruction would the nurse include when teaching a pregnant client with class II heart disease? A.) Advise her to gain at least 30 pounds B.) Instruct her to avoid strenuous activity C.) Inform her of the need to limit fluid intake D.) Explain the importance of a diet high in calcium

B

Which is a danger sign of pregnancy? A.) Constipation B.) Alteration in the pattern of fetal movement C.) Heart palpitations D.) Edema in the ankles and feet at the end of the day

B

Which is the expected total weight gain for the client with a singleton pregnancy during the first trimester? A.) 2 to 3 kg B.) 1 to 2 kg C.) 1.5 to 2 kg D.) 0.5 to 1 kg

B

Which is the normal range of amniotic fluid index? A.) 1 to 5 cm B.) 10 to 25 cm C.) 25 to 40 cm D.) 40 to 65 cm

B

Which nursing action is the primary step of assessment when meeting with a new client to discuss contraception? A.) Obtain data about the frequency of coitus B.) Determine the woman's level of knowledge about contraception and commitment to any particular method C.) Assess the woman's willingness to touch her genitals and cervical mucus D.) Evaluate the woman's contraceptive life plan

B

Which nursing explanation is appropriate for a client who asks about decreasing the risk for transmission of human immunodeficiency virus (HIV) to her baby during baby? A.) It is recommended that the baby be delivered by cesarean section B.) If the viral load is decreased and you are treated in the intrapartum period, the risk of transmission is reduced C.) If your viral load is low and you recieve zidovudine during labor, there is minimal risk of your baby contracting HIV D.) To decrease the risk of transmitting HIV to the baby; it is essential that you receive adequate treatment during labor

B

Which nursing explanation is appropriate for a client who is estimated to be at 6 weeks of gestation by ultrasonography and states, "I don't understand how I can be pregnant; I just had my period?" A.) The embryo has not reached the uterine lining triggering menses B.) The bleeding was a result of the embryo attaching to the uterine lining C.) The levels of estrogen and progesterone are decreased after conception D.) Some women still experience menses during during the initial period of conception

B

Which nursing explanation is appropriate to include when preparing a client for a nonstress test? A.) "I will be using stimulation to wake up the baby" B.) "You will recline a bit with a slight tilt to the side' C.) "Push this button when you feel the baby move" D.) "You can lie on your back and get comfortable"

B

Which nursing information is appropriate to include when preparing to teach a client about calcium intake and pregnancy? A.) Fetal growth increases the need for calcium B.) Dietary intake of calcium is generally inadequate C.) Calcium supplementation is necessary during pregnancy D.) The majority of people are unable to digest foods that are high in calcium

B

Which nursing information is appropriate to include when preparing to teaching a client about calcium intake and pregnancy? A.) Fetal growth increases the need for calcium B.) Dietary intake of calcium is generally inadequate C.) Calcium supplementation is necessary during pregnancy D.) The majority of people are unable to digest foods that are high in calcium

B

Which nursing information is needed to understand and guide a client through her acceptance of pregnancy? A.) Nonacceptance of the pregnancy very often equates to rejection of the child B.) Mood swings most likely are the result of worries about finances and a changed lifestyle, as well as profound hormonal changes C.) Ambivalent feelings during pregnancy usually are seen only in emotionally immature or very young mothers D.) Conflicts such as not wanting to be pregnant or childrearing and career-related decisions should not be addressed during pregnant because they will resolve themselves naturally after birth

B

Which nursing instruction is appropriate for the pregnant client experiencing nausea and vomiting? A.) Drink a glass of water with a fat-free carbohydrate before getting out of bed in the morning B.) Eat small, frequent meals (every 2 to 3 hours) C.) Increase intake of high-fat foods to keep the stomach full and coated D.) Limit fluid intake throughout the day

B

Which nursing intervention is necessary before a first-trimester transabdominal ultrasound? A.) Place the woman on nothing by mouth (nil per os [NPO]) for 12 hours B.) Instruct the woman to drink 1 to 2 quarts of water C.) Administer an enema. D.) Perform an abdominal preparation

B

Which physiologic system would be appropriate to evaluate with laboratory testing for a client who is at 37 weeks of gestation and is having itching all over her body? A.) Renal B.) Hepatic C.) Immune D.) Endocrine

B

Which question is appropriate for a client at 36 weeks of gestation who is having problems with her legs swelling and has developed hemorrhoids? A.) Are you taking iron supplements B.) Do you have any pain in your legs C.) Have you been experiencing frequent constipation D.) Have you tried adding foods with fiber in your daily diet

B

Which sign of a potential complication is the most important for the nurse to share with the client? A.) Constipation B.) Alteration in the pattern of fetal movement C.) Heart palpitations D.) Edema in the ankles and feet at the end of the day

B

Which statement best describes the rationale for the physiologic anemia that occurs during pregnancy? A.) Physiologic anemia involves an inadequate intake of iron B.) Dilution of hemoglobin concentration occurs in pregnancy with physiologic anemia C.) Fetus establishes the iron stores D.) Decreased production of erythrocytes occur

B

Which statement regarding the probable signs of pregnancy is most accurate? A.) Determined by ultrasound B.) Observed by the health care provider C.) Reported by the client D.) Confirmed by diagnostic tests

B

Which time-based description of a stage of development in pregnancy is correct? A.) Viability: 22 to 37 weeks of gestation since the last menstrual period (assuming a fetal weight greater than 500 g) B.) Term: pregnancy from the beginning of 38 weeks of gestation to the end of 42 weeks of gestation C.) Preterm: pregnancy from 20 to 28 weeks of gestation D.) Postdate: pregnancy that extends beyond 38 weeks of gestation

B

The nurse is caring for a diabetic client who is pregnant. Which education regarding self care during illness would the nurse provide the client (select all that apply): A.) Avoid insulin if your appetite is less than normal B.) Drink as much fluid as possible C.) Obtain as much rest as possible D.) Check blood glucose levels at regular intervals E.) Seek emergency treatment if your glucose level exceeds 250 mg/dL

B, C, D

Which client's with diabetes may develop complications if they perform exercises (select all that apply): A.) A client who is on insulin B.) A client with diabetic ketoacidosis C.) A client with uncontrolled hypertension D.) A client with severe peripheral neuropathy E.) A client who has lost 5 kg weight after diagnosis

B, C, D

Which fetal abnormalities are prevented by monitoring an obstetric client with phenylketonuria (PKU) with high levels of phenylalanine (select all that apply): A.) Hydrocephaly B.) Cardiac anomalies C.) Intellectual disabilities D.) Intrauterine growth restriction E.) Hypopigmentation of the skin

B, C, D

Which information would the nurse include in the preconception teaching for a client with Marfan syndrome who presents aortic root diameter of more than 6 cm (select all that apply): A.) It should not be repaired before you become pregnant B.) There is an increased chance of maternal mortality C.) The newborn child may develop Marfan syndrome D.) You may not be able to deliver the child vaginally E.) You should not even try to become pregnant at all

B, C, D

Which statements about genital herpes are accurate (select all that apply): A.) Genital herpes is also known as genital warts B.) Stress, menstruation, trauma and illness have been known to trigger recurrences C.) Genital herpes is chronic and recurring, and it has no known cure D.) Plain soap and water are all that are needed to clean hands that have come in contact with herpetic lesions E.) Contact isolation is needed for clients with genital herpes

B, C, D

Autoimmune disorders often occur during pregnancy because a large percentage of women with an autoimmune disorder are of childbearing age. Which disorders fall into the category of collagen vascular disease (select all that apply): A.) Multiple sclerosis B.) SLE C.) Antiphospholipid syndrome D.) Rheumatoid arthritis E.) Myasthenia gravis

B, C, D, E

Foodborne illnesses can cause adverse effects for both mother and fetus. The nurse is in an ideal position to evaluate the client's knowledge regarding steps to prevent a foodborne illness. The nurse asks the client to teach back the fours simple steps of food preparation. What are they (select all that apply): A.) Purchase B.) Clean C.) Separate D.) Cook E.) Chill

B, C, D, E

Which behaviors place women at high risk for human immunodeficiency virus (HIV) (select all that apply): A.) Abstinence B.) IV drug use C.) Having multiple sex partners D.) Engaging in high-risk sexual behaviors E.) Having a history of multiple sexually transmitted infections (STIs)

B, C, D, E

In providing teaching to a pregnant client, the nurse knows which are some common maternal and fetal indications for antepartum testing (select all that apply): A.) Gallstones B.) Preeclampsia C.) Previous stillbirth D.) Fetal growth restriction E.) Increased fetal movement F.) Premature rupture of membranes

B, C, D, F

A nonstress test (NST) is ordered on a pregnant women at 37 weeks gestation. Which are the most appropriate teaching points to include when explaining the procedure to the client (select all that apply): A.) After 20 minutes, a nonreactive reading indicates the test is complete B.) Vibroacoustic stimulation may be used during the test C.) Drinking orange juice before the test is appropriate D.) A needle biopsy may be needed to stimulate contractions E.) Two sensors are placed on the abdomen to measure contractions and fetal heart tones

B, C, E

A pregnant client with pregestational insulin-dependent diabetes is going for a week's vacation in another state. What would the nurse ask the client to carry with her to prevent complications (select all that apply): A.) Antibiotics B.) Insulin vials C.) Glucose tablets D.) Antihypertensives E.) Blood glucose meter

B, C, E

The client and her partner are considering male sterilization as a form of permanent birth control. While educating the client regarding the risks and benefits of the procedure, which information should the nurse include (select all that apply): A.) Sterilization should be performed under general anesthesia B.) Pain, bleeding, and infection are possible complications C.) Pregnancy may still be possible D.) Vasectomy may affect potency E.) Secondary sex characteristics are unaffected

B, C, E

Which statements regarding the HPV are accurate (select all that apply): HPV infections: A.) Are thought to be less common in pregnant women than in women who are not pregnant B.) Are thought to be more common in pregnant women than in women who are not pregnant C.) Were previously called genital warts D.) Were previously called herpes E.) May cause cancer

B, C, E

The number of routine laboratory tests during follow-up visits is limited; however, those that are performed are essential. Which statements regarding group B Streptococcus (GBS) testing are correct (select all that apply): A.) Performed between 32 and 34 weeks of gestation B.) Performed between 35 and 37 weeks of gestation C.) All women should be tested D.) Only women planning a vaginal birth should be tested E.) Women with a history of GBS should be retested

B, D, E

Which foods are appropriate to exclude from the pregnant client's diet plan to ensure good health (select all that apply): A.) Meat B.) Butter C.) Yogurt D.) Beef fat E.) Stick margarine

B, D, E

Which foods are appropriate to prevent calcium imbalance when replacing milk for a pregnant client (select all that apply): A.) Rice B.) Cocoa C.) Carrots D.) Yogurt D.) Buttermilk

B, D, E

Which foods are appropriate to suggest to prevent calcium deficiency for the client who maintains a vegan diet (select all that apply): A.) Cheese B.) Collards C.) Carrots D.) Dried figs E.) Cooked dried beans

B, D, E

Which foods would the nurse recommend that the client avoid during pregnancy (select all that apply): A.) Hot dogs B.) Brie cheese C.) Luncheon meals D.) Unpasteurized milk E.) Deli made egg salade

B, D, E

A woman who has just undergone a first-trimester abortion will be using oral contraceptives. To protect against pregnancy, the client should be advised to do what? A.) Avoid sexual contact for at least 10 days after starting the pill B.) Use condoms and foam for the first few weeks as a backup C.) Use another method of contraception for 1 week after starting the pill D.) Begin sexual relations once vaginal bleeding has ended

C

A woman will be taking oral contraceptives using a 28-day pack. What advice should the nurse provide to protect this client from an unintended pregnancy? A.) Limit sexual contact for one cycle after starting the pill B.) Use condoms and foam instead of the pill for as long as the client takes an antibiotic C.) Take one pill at the same time every day D.) Throw away the pack and use a backup method if two pills are missed during week 1 of her cycle

C

A woman with asthma is experiencing a postpartum hemorrhage. Which drug should be avoided when treating postpartum bleeding to avoid exacerbating asthma? A.) Oxytocin (Pitocin) B.) Nonsteroidal antiinflammatory drugs (NSAIDs) C.) Hemabate D.) Fentanyl

C

At 35 weeks of pregnancy, a woman experiences preterm labor. Although tocolytic medications are administered and she is placed on bed rest, she continues to experience regular uterine contractions and her cervix is beginning to dilate and efface. What is an important test for fetal well-being at this time? A.) PUBS B.) Ultrasound for fetal size C.) Amniocentesis for fetal lung maturity D.) NST

C

Dental care during pregnancy is an important component of good prenatal care. Which instruction regarding dental health should the nurse provide? A.) Regular brushing and flossing may not be necessary during early pregnancy because it may stimulate the woman who is already nauseated to vomit. A cleaning is all that is necessary B.) Dental surgery, in particular, is contraindicated during pregnancy and should be delayed until after delivery C.) If dental treatment is necessary, then the woman will be most comfortable with it in the second trimester D.) If a woman has dental anxiety, then dental care may interfere with the expectant mothers need to practice conscious relaxation and to prepare for labor

C

During a prenatal visit, the nurse is explaining dietary management to a woman with pregestational diabetes. Which statement by the client reassures the nurse that teaching has been effective? A.) I will need to eat 600 more calories per day because I am pregnant B.) I can continue with the same diet as before pregnancy as long as it is well balanced C.) Diet and insulin needs change during pregnancy D.) I will plan my diet based on the results of urine glucose testing

C

In which pregnant client would the nurse identify the need to screen for undiagnosed homozygous maternal phenylketonuria (PKU)? A.) A client who had a macrosomic fetus in a previous pregnancy B.) A client who had obstructed labor in a previous pregnancy C.) A client who gave birth to a microcephalic infant in a previous pregnancy D.) A client who had placental insufficiency in a previous pregnancy

C

Some pregnant clients may complain of changes in their voice and impaired hearing. What should the nurse explain to the client concerning these findings? A.) Voice changes are caused by decreased estrogen levels B.) Displacement of the diaphragm results in thoracic breathing C.) Voice changes and impaired hearing are due to the results of congestion and swelling of the upper respiratory tract D.) Increased blood volume causes changes in the voice

C

The nurse is assessing a client for gestational diabetes mellitus (GDM) using the oral glucose tolerance test (OGTT). What intervention by the nurse is appropriate when caring for this client? A.) Teach the client to eat an unrestricted diet the day before the test B.) Instruct the client to avoid caffeine for 6 hours before the test C.) Draw blood for a fasting blood glucose level just before the test D.) Obtain the plasma glucose level 1 hour after a 50-g oral glucose load

C

The nurse is assessing a client with hyperemesis gravidarum during the early stages of pregnancy. Which nonpharmacologic measure is appropriate to alleviate the discomforts associated with nausea and vomiting? A.) Having the client cook her favorite foods B.) Allowing frequent visits from friends C.) Providing environment that is free from odors D.) Having the client eat warm, low-fat, soupy foods

C

The nurse is assessing a pregnant client with a history of atrial septal defect. The nurse knows that the client is at risk for right-sided heart failure because of which factor? A.) Ruptured aorta B.) Stiffened valve leaflets C.) Increased plasma volume D.) Increased pressure in pulmonary veins

C

The nurse is caring for a diabetic client who is breastfeeding her infant. Within which time frame after childbirth would the client's insulin requirements return to prepregnancy levels? A.) Immediately after childbirth B.) Seven to ten days after childbirth C.) On completion of weaning D.) During the lactation period

C

The nurse is caring for a pregnant client who is scheduled for cordocentesis. Which could a complication of the test? A.) Destruction of red blood cells B.) Fetal hyperbilirubinemia C.) Fetomaternal hemorrhage D.) Deformity of extremities

C

The nurse is providing contraceptive instruction to a young couple who are eager to learn. The nurse should be cognizant of which information regarding the natural family planning method? A.) The natural family planning method is the same as coitus interruptus or pulling out B.) This contraception method uses the calendar method to align the woman's cycle with the natural phases of the moon C.) This practice is the only contraceptive method acceptable to the Roman Catholic Church D.) The natural family planning method relies on barrier methods during the fertility phases

C

What is the primary role of the nonpregnant partner during pregnancy? A.) To provide financial support B.) To protect the pregnant woman from old wives tales C.) To support and nurture the pregnant woman D.) To make sure the pregnant woman keeps prenatal appointments

C

What should the nurse be cognizant of concerning the clients reordering of personal relationships during pregnancy? A.) Because of the special motherhood bond, a woman's relationship with her mother is even more important than with the father of the child B.) Nurses need not get involved in any sexual issues the couple has during pregnancy, particularly if they have trouble communicating them to each other C.) Women usually express two major relationship needs during pregnancy: feeling loved and valued and having the child accepted by the father D.) The woman's sexual desire is likely to be highest in the first trimester because of the excitement and because intercourse is physically easier

C

When assessing a pregnant client, the nurse is aware of which maternal and neonatal risks associated with gestational diabetes mellitus? A.) Maternal premature rupture of membranes and neonatal sepsis B.) Maternal hyperemesis and neonatal low birth weight C.) Maternal preeclampsia and fetal macrosomia D.) Maternal placenta previa and fetal prematurity

C

When would a client using the calendar rhythm method for contraception abstain from sexual activity to avoid pregnancy? A.) On day 9 B.) On day 15 C.) From days 10 to 17 D.) From days 2 to 10

C

Which action by the pregnant client demonstrates understanding of the nurse's instructions regarding relief of leg cramps? A.) Wiggles and points her toes during the cramp B.) Applies cold compresses to the affected leg C.) Extends her leg and dorsiflexes her foot during the cramp D.) Avoids weight bearing on the affected leg during the cramp

C

Which action would the nurse take if a client with suspected syphilis has a negative nontreponemal antibody test result? A.) Assess the client's medical history B.) Assess for the presence of genital lesions C.) Administer the test again after 1 to 2 months D.) Assess if the client is receiving alternative therapy

C

Which client would be an ideal candidate for injectable progestins such as Depo-Provera (DMPA) as a contraceptive choice? A.) The ideal candidate for DMPA wants menstrual regularity and predictability B.) The client has a history of thrombotic problems or breast cancer C.) The ideal candidate has difficulty remembering to take oral contraceptives daily D.) The client is homeless or mobile and rarely receives health care

C

Which consideration is essential for the nurse to understand regarding follow-up prenatal care visits? A.) The interview portions become more intensive as the visits become more frequent over the course of the pregnancy B.) Monthly visits are scheduled for the first trimester, every 2 weeks for the second trimester, and weekly for the third trimester C.) During the abdominal examination, the nurse should be alert for supine hypotension D.) For pregnant women, a systolic BP of 130 mm Hg and a diastolic BP of 80 mm Hg is sufficient to be considered hypertensive

C

Which contraceptive method is contraindicated in woman with a history of toxic shock syndrome? A.) Condom B.) Spermicide C.) Cervical cap D.) Oral contraceptives

C

Which dietary finding is the rationale for the nursing instruction to the pregnant client to eat nuts, legumes, cocoa and whole grains during the second trimester? A.) A diet that is low in sinz B.) Low intake of vitamin A C.) Low intake of magnesium D.) Decreased vitamin D intake

C

Which explains to the pregnant client the reason for constipation in the second trimester? A.) Progesterone levels decrease gastric acid secretions B.) Progesterone levels increase gastrointestinal (GI) mostly in pregnant women C.) Iron supplements may cause constipation and darkened stool D.) Constipation is caused by inadequate carbohydrate intake

C

Which explanation is appropriate when a pregnant client has low hematocrit values but is not considered anemic? A.) Hematocrit does not related to anemia B.) Anemia does not cause risk during pregnancy C.) Plasma volume expands rapidly during pregnancy D.) Erythrocyte production compensates for the low hematocrit

C

Which hematocrit (HCT) and hemoglobin (HGB) results represent the lowest acceptable values for a woman in the third trimester of pregnancy? A.) 38% HCT; 14 g/dl HGB B.) 35% HCT; 13 g/dl HGB C.) 33% HCT; 11 g/dl HGB D.) 32% HCT; 10.5 g/dl HGB

C

Which important component of nutritional counseling should the nurse include in health teaching for a pregnant woman who is experiencing cholecystitis? A.) Assess the woman's dietary history for adequate calories and proteins B.) Teach the woman that the bulk of calories should come from proteins C.) Instruct the woman to eat a low-fat diet and to avoid fried foods D.) Instruct the woman to eat a low-cholesterol, low-salt diet

C

Which information should nurses provide to expectant mothers when teaching them how to evaluate daily fetal movement counts (DFMCs)? A.) Alcohol or cigarette smoke can irritate the fetus into greater activity B.) Kick counts should be taken every hour and averaged every 6 hours, with every other 6-hour stretch off C.) The fetal alarm signal should go off when fetal movements stop entirely for 12 hours D.) A count of less than four fetal movements in 1 hour warrants future evaluation

C

Which intervention would be the most beneficial to a pregnant client with cervical Chlamydia trachomatis? A.) Administering doxycycline 100 mg twice a day B.) Administering silver nitrate solution on a regular basis C.) Administering azithromycin 1 g orally once a day D.) Administering human papillomavirus (HPV) vaccine

C

Which is appropriate client knowledge when using the basal body temperature method of family planning? A.) She will remain fertile for 5 days after ovulation B.) She should take her temperature each night before going to bed C.) Her temperature will increase about 0.4 to 0.8 F after ovulation D.) Her temperature is normally lower during the second half of her cycle

C

Which is the expected delivery date for a pregnant woman whose first day of her last menstrual period was April 20, 2019? A.) December 27, 2019 B.) January 20, 2020 C.) January 27, 2020 D.) February 7, 2020

C

Which is the role of the nurse in initial family planning? A.) Refer the couple to a reliable health care provider B.) Decide on the best method for the couple C.) Educate couples on the various methods of contraception D.) Advise couples on which contraceptive to use

C

Which lubricant used by the client reduces contraceptive effect? A.) Nonoxynol-9 lubricant B.) Silicon-based lubricant C.) Suntan oil as lubricant D.) Water-soluble lubricant

C

Which minerals and vitamins are recommended to supplement the pregnant client's diet? A.) Fat-soluble vitamins A and D B.) Water-soluble vitamins C and B6 C.) Iron and folate D.) Calcium and zinc

C

Which minerals and vitamins are usually recommended as a supplement in a pregnant clients diet? A.) Fat-soluble vitamins A and D B.) Water-soluble vitamins C and B6 C.) Iron and folate D.) Calcium and zinc

C

Which neurologic condition would require preconception counseling, if at all possible? A.) Eclampsia B.) Bell palsy C.) Epilepsy D.) Multiple sclerosis

C

Which nursing advice is appropriate for a 5-month pregnant client reporting dizziness after waking up in the morning? A.) "Keep your legs elevated while sleeping" B.) "Try to spend less of your time sleeping" C.) "Try sleeping in the side-lying (lateral) position" D.) "Use two pillows for your head while sleeping"

C

Which nursing advice is appropriate for a client who is taking combined oral contraceptives (COCs) and complains of increased appetite and post-pill amenorrhea? A.) "Take herbal supplements to prevent these side effects" B.) "Take the pills at the same time each day" C.) "The primary health care provide can prescribe a COC with a low dose of hormone" D.) "These side effects are temporary and will diminish soon"

C

Which nursing assessment finding is anticipated for a pregnant client with a history of poor nutritional intake? A.) Poor skin turgor B.) Erosion of dental enamel C.) Hyperactive bowel sounds D.) Yellow discharge from eyes

C

Which nursing conclusion about a 6-month primigravida is appropriate when blood tests report: - Hemoglobin: 11 g/dL - RBC: 5.5 million/mm3 - Hematocrit: 33% - WBC: 12,000/mm3 A.) Iron deficiency B.) At risk of bleeding C.) Physiologic anemia D.) Myelosuppression

C

Which nursing explanation is appropriate for a client who has type 1 diabetes and at 36 weeks of gestation asks why her insulin requirement has increased throughout the pregnancy? A.) The metabolic rate in pregnancy has increased requiring additional insulin B.) The rapid growth of the fetus has resulted in an increased demand for insulin C.) The hormonal levels associated with pregnancy have increased your resistance to insulin D.) The supplementary level of insulin is required to manage the additional intake of carbohydrates

C

Which nursing information is appropriate for a client who uses the vaginal contraceptive ring, but after feeling discomfort during coitus removed the ring for 1 hour and is now concerned that removal could have increased the chance of pregnancy? A.) "You need to be tested for pregnancy" B.) "The ring will lose efficacy if removed often" C.) "There is no risk if the ring is reinserted in 3 hours" D.) "It may increase the risk of sexually transmitted infection"

C

Which nursing information is appropriate for a pregnant client at 10 weeks of gestation who jogs three or four times per week and is concerned about the effect of exercise of the fetus? A.) You don't need to modify your exercising any time during your pregnancy B.) Stop exercising because it will harm the fetus C.) You may find that around the seventh month of your pregnancy, you will need to modify your exercise to walking D.) Jogging is too hard on your joints; switch to walking now

C

Which nursing instruction is appropriate for the pregnant client taking oral iron supplements and experiencing constipation? A.) Drink mineral oil before going to bed B.) Take a stool softened before going to bed C.) Drink six to eight glasses of water every day D.) Discontinue taking iron supplements

C

Which nursing response is appropriate for the expectant father who confides in the nurse that his 10-week-pregnant wife is driving him crazy seeming happy one minute and crying over nothing the next minute? A.) This is normal behavior and should begin to subside by the second trimester B.) She may have difficulty adjusting to pregnancy C.) This is called emotional lability and is related to hormonal changes and anxiety during pregnancy. The mood swings will eventually subside as she adjusts to being pregnant D.) You seem impatient with her. Perhaps this is precipitating her behavior

C

Which preexisting factor is known to increase the risk of GDM? A.) Underweight before pregnancy B.) Maternal age younger than 25 years C.) Previous birth of large infant D.) Previous diagnosis of type 2 diabetes mellitus

C

Which sexually transmitted infection (STI) is the most commonly reported in American women? A.) Gonorrhea B.) Syphilis C.) Chlamydia D.) Candidiasis

C

Which sexually transmitted infection would the nurse suspect for a client who reports purulent anal discharge, rectal pain and blood in her stool? A.) Syphilis B.) Chlamydia C.) Gonorrhea D.) Human papillomavirus (HPV)

C

Which statement accurately describes the centering model of care? A.) Group sessions begin with the first prenatal visit B.) Blood pressure (BP), weight, and urine dipsticks are obtained by the nurse at each visit C.) Approximately 8 to 12 women are placed in each gestational-age cohort group D.) Outcomes are similar to traditional prenatal care

C

Which statement by the client indicates a risk of genital tract infection? A.) I eat yogurt almost every day B.) I use hosiery with a cotton crotch C.) I take a bubble bath once a day D.) I void before and after intercourse

C

Which statement by the client with gestational diabetes indicates an understanding of the information provided by the nurse? A.) "I will limit my intake of alcohol" B.) "I will use artificial sweeteners instead of sugar in my coffee" C.) "I will not go more than 4 hours throughout the day without eating" D.) "I will eat a large dinner so I do not become hypoglycemic during the night"

C

Which statement describes the recommended weight gain and nutritional goals for the adolescent client who is pregnant? A.) "The caloric intake is increased for adolescents" B.) "Recommended weight goals are different for adolescents" C.) "The nutritional intake for adolescents is the same as for adults" D.) "Recommend the weight gain goal on the upper end of the adolescent BMI chart"

C

Which statement regarding the laboratory test for glycosylated hemoglobin Alc is correct? A.) The laboratory test for glycosylated hemoglobin Alc is performed for all pregnant women, not only those with or likely to have diabetes B.) This laboratory test is a snapshot of glucose control at the moment C.) This laboratory test measures the levels of hemoglobin Alc, which should remain at less than 7% D.) This laboratory test is performed on the woman's urine, not her blood

C

Which symptom for a client taking an oral contraceptive pill (OCP) as her birth control method of choice requires immediate health care provider attention? A.) Breast tenderness and swelling B.) Weight gain C.) Swelling and pain in one of her legs D.) Mood swings

C

Which symptoms in the pregnant client would prompt the nurse to immediately inform the primary health care provider? A.) Dyspnea B.) Persistent anemia C.) Fluid leaking from the vagina D.) Imbalanced nutrition

C

Which testing would be available for a client at 11 weeks of gestation who requests a fetal genetic assessment? A.) Ultrasonography B.) Amniocentesis C.) Chorionic villus sampling D.) Percutaneous umbilical blood sampling (PUBS)

C

Which type of cervical mucus would you expect when the woman is ovulating? A.) Scant B.) Thick, cloudy and sticky C.) Clear, wet, sticky and slippery D.) Cloudy, yellow or white, and sticky

C

While obtaining a diet history, the nurse might be told that the expectant mother has cravings for ice chips, cornstarch, and baking soda. Which nutritional problem does this behavior indicate? A.) Preeclampsia B.) Pyrosis C.) Pica D.) Purging

C

With regard to weight gain during pregnancy, the nurse should be aware of which important information? A.) In pregnancy, the woman's height is not a factor in determining her target weight B.) Obese women may have their health concerns, but their risk of giving birth to a child with major congenital defects is the same as with women of normal weight C.) Women with inadequate weight gain have an increased risk of delivering a preterm infant with intrauterine growth restriction (IUGR) D.) Greater than expected weight gain during pregnancy is almost always attributable to old-fashioned overeating

C

The nurse is teaching a client how to monitor blood glucose using a glucose meter. Arrange the steps of this procedure into the correct order for the client to perform: A.) Record the results displayed on the meter B.) Select a site on the side of a finger C.) Wash hands with warm water D.) Gently squeeze the finger E.) Pierce the site with a lancelet F.) Let blood be drawn into the test strip

C, B, E, D, F, A

A client is diagnosed with mitral valve stenosis. Because of this diagnosis, the nurse is aware that the client has which increased risks (select all that apply): A.) Hemorrhage B.) Thrombophelbitis C.) Pulmonary edema D.) Pulmonary embolism E.) Right-sided heart failure

C, D, E

A pregnant client with cystic fibrosis (CF) wants to breastfeed her infant. Which assessments would be performed to make sure that breastfeeding will be safe and effective? A.) Monitor maternal weight B.) Monitor maternal urine for ketones C.) Monitor sodium levels in breast milk D.) Monitor total fat levels in breast milk E.) Monitor the infant growth pattern

C, D, E

Hypothyroidism occurs in 2 to 3 pregnancies per 1000. Because severe hypothyroidism is associated with infertility and miscarriage, it is not often seen in pregnancy. Regardless of this fact, the nurse should be aware of the characteristic symptoms of hypothyroidism. Which do they include (select all that apply): A.) Hot flashes B.) Weight loss C.) Lethargy D.) Decrease in exercise capacity E.) Cold intolerance

C, D, E

Which conditions describe the assessment findings for a client with secondary syphilis (select all that apply): A.) Papules B.) Chancre C.) Lymphadenopathy D.) Condylomata lata on the vulva, perineum, or anus E.) Maculopapular rash on the hands and soles of the feet

C, D, E

Which dietary modifications are appropriate for the pregnant client who has a folate intake of approximately 580 mcg/day (select all that apply): A.) Include 8 ounces of milk daily B.) Include 6 ounces of yogurt daily C.) Add one extra slice of bread daily D.) Include one boiled egg every day E.) Include one-half cup of corn daily

C, D, E

Which findings obtained during clinical evaluation of a pregnant client help determine the gestational age of the fetus (select all that apply): A.) Previous cesarean delivery B.) Types of contraception used C.) Current fundal height D.) Current week of gestation

C, D, E

Which infections are collectively known as TORCH infections (select all that apply): A.) Chlamydia B.) Gonorrhea C.) Toxoplasmosis D.) German measles E.) Cytomegalovirus F.) Herpes genitalis

C, D, E, F

A female client tells the nurse that her male partner prefers not to use condoms during intercourse. The client is worried that if she requests that he use a condom, her partner may be offended. Which advice would the nurse provide the client? A.) You will suffer terrible consequences if you don't use a condom B.) You can have unprotected sex once in a while, but use a condom often C.) Carry condoms with you, and demand that your partner use one if he wants to have sex D.) Discuss the importance of using condoms at a time when you are not having sex

D

A pregnant client at 28 weeks of gestation has been diagnosed with gestational diabetes. Which nursing action is appropriate when caring for this client? A.) Oral hypoglycemic agents can be used if the client is reluctant to give herself insulin B.) Dietary modifications and insulin are both required for adequate treatment C.) Glucose levels are monitored by testing urine four times a day and at bedtime D.) Dietary management involved disturbing nutrient requirements over three meals and two or three snacks

D

A pregnant client has atypical sharp pain in the left side of the chest that does not respond to nitrates. Which diagnosis would the nurse expect in this client? A.) Tetralogy of Fallot B.) Atrial septal defect C.) Mitral valve stenosis D.) Mitral valve prolapse

D

A pregnant couple has formulated a birth plan and is reviewing it with the nurse at an expectant parents class. Which aspect of their birth plan should be considered potentially unrealistic and require further discussion with the nurse? A.) My husband and I have agreed that my sister will be my coach because he becomes anxious with regard to medical procedures and blood. He will be nearby and check on me every so often to make sure everything is okay B.) We plan to use the techniques taught in the Lamaze classes to reduce the pain experienced during labor C.) We want the labor and birth to take place in a birthing room. My husband will come in the minute the baby is born D.) Regardless of the circumstances, we do not want the fetal monitor used during labor because it will interfere with movement and doing effleurage

D

A pregnant woman at 33 weeks of gestation is brought to the birthing unit after a minor automobile accident. The client is experiencing no pain and no vaginal bleeding, her vital signs are stable, and the FHR is 132 beats per minute with variability. What is the nurses highest priority? A.) Monitoring the woman for a ruptured spleen B.) Obtaining a physician's order to discharge her home C.) Monitoring her for 24 hours D.) Using continuous EFM for a minimum of 4 hours

D

A pregnant woman's diet may not meet her increased need for folates. Which food is a rich source of this nutrient? A.) Chicken B.) Cheese C.) Potatoes D.) Green leafy vegetables

D

A woman arrives at the clinic seeking confirmation that she is pregnant. The following information is obtained: She is 24 years old with a body mass index (BMI) of 17.5. She admits to having used cocaine several times during the past year and occasionally drinks alcohol. Her blood pressure is 108/70 mm Hg. The family history is positive for diabetes mellitus and cancer. Her sister recently gave birth to an infant with a neural tube defect (NTD). Which characteristics places this client in a high-risk category? A.) Blood pressure, age, BMI B.) Drug and alcohol use, age, family history C.) Family history, blood pressure (BP), BMI D.) Family history, BMI, drug and alcohol abuse

D

A woman has come to the clinic for preconception counseling because she wants to start trying to get pregnant. Which guidance should she expect to receive? A.) Discontinue all contraception now B.) Lose weight so that you can gain more during pregnancy C.) You may take any medications you have been regularly taking D.) Make sure you include adequate folic acid in your diet

D

A woman in the 34th week of pregnancy reports that she is very uncomfortable because of heartburn. Which recommendation would be appropriate for this client? A.) Substitute other calcium sources for milk in her diet B.) Lie down after each meal C.) Reduce the amount of fiber she consumes D.) Eat five small meals daily

D

A woman is 3 months pregnant. At her prenatal visit she tells the nurse that she does not know what is happening; one minute she is happy that she is pregnant and the next minute she cries for no reason. Which response by the nurse is most appropriate? A.) Don't worry about it; you'll feel better in a month or so B.) Have you talked to your husband about how you feel? C.) Perhaps you really don't want to be pregnant D.) Hormone changes during pregnancy commonly result in mood swings

D

A woman is using the basal body temperature (BBT) method of contraception. She calls the clinic and tells the nurse, My period is due in a few days, and my temperature has not gone up. What is the nurses most appropriate response? A.) This probably means that you're pregnant B.) Don't worry; it's probably nothing C.) Have you been sick this month? D.) You probably didn't ovulate during this cycle

D

A woman who is 16 weeks pregnant has come in for a follow-up visit with her significant other. To reassure the client regarding fetal well-being, which is the highest priority action for the nurse to perform? A.) Assess the fetal heart tones with a Doppler stethoscope B.) Measure the girth of the woman's abdomen C.) Complete an ultrasound examination (sonogram) D.) Offer the woman and her family the opportunity to listen to the fetal heart tones

D

A woman who is at 36 weeks of gestation is having a nonstress test. Which statement indicates her correct understanding of the test? A.) "I will need to have a full bladder for the test to be done accurately" B.) "I should have my husband drive me home after the test because I may be nauseated" C.) "This test will help to determine if the baby has Down Syndrome or a neural tube defect" D.) "This test observes for fetal activity and an acceleration of the fetal heart rate to determine the well-being of the baby"

D

A woman with gestational diabetes has had little or no experience reading and interpreting glucose levels. The client shows the nurse her readings for the past few days. Which reading signals the nurse that the client may require an adjustment of insulin or carbohydrates? A.) 75 mg/dl before lunch. This is low; better eat now B.) 115 mg/dl 1 hour after lunch. This is a little high; maybe eat a little less next time C.) 115 mg/dl 2 hours after lunch. This is too high; it is time for insulin D.) 50 mg/dl just after waking up from a nap. This is too low; maybe eat a snack before going to sleep

D

An adolescent complains of postcoital bleeding and purulent cervical discharge. On assessment, the nurse finds that the adolescent has multiple sex partners. What is the priority nursing action in this case? A.) Encourage the client to use a condom B.) Obtain a thorough history of allergies C.) Recommend that the client undergo yearly screening for sexually transmitted infections (STIs) D.) Prepare the client for STI testing

D

An adolescent complains of postcoital bleeding and purulent cervical discharge. On assessment, the nurse finds that the adolescent has multiple sex partners. What is the priority nursing action in this case? A.) Encourage the client to use a condom B.) Obtain a thorough history of allergies C.) Recommend that the client undergo yearly screening for sexually transmitted infections (STIs) D.) Preparing the client for STI testing

D

Another common pregnancy-specific condition is pruritic urticarial papules and plaques of pregnancy (PUPPP). A client asks the nurse why she has developed this condition and what can be done. What is the nurse's best response? A.) PUPPP is associated with decreased maternal weight gain B.) The rate of hypertension decreases with PUPPP C.) This common pregnancy-specific condition is associated with a poor fetal outcome D.) The goal of therapy is to relieve discomfort

D

During a physical assessment of an at-risk client, the nurse notes generalized edema, crackles at the base of the luns and some pulse irregularity. These are most likely signs of which problems? A.) Euglycemia B.) Rheumatic fever C.) Pneumonia D.) Cardiac decompensation

D

During a prenatal check-up a client who is 7 months pregnant reports that she is able to feel about two kicks in an hour. The nurse refers the client for an ultrasound, and which is the primary reason for this referral? A.) To check for fetal anomalies B.) To check gestational age C.) To check fetal position D.) To check for fetal well-being

D

Group B Streptococcus (GBS) is part of the normal vaginal flora in 20% to 30% of healthy pregnant women. GBS has been associated with poor pregnancy outcomes and is an important factor in neonatal morbidity and mortality. Which finding is not a risk factor for neonatal GBS infection? A.) Positive prenatal culture B.) Preterm birth at 37 weeks or less of gestation C.) Maternal temperature of 38 C or higher D.) Premature rupture of membranes (PROM) 24 hours or longer before the birth

D

In comparing the abdominal and transvaginal methods of ultrasound examination, which information should the nurse provide to the client? A.) Both require the woman to have a full bladder B.) The abdominal examination is more useful in the first trimester C.) Initially, the transvaginal examination can be painful D.) The transvaginal examination allows pelvic anatomy to be evaluated in greater detail

D

In her work with pregnant women of different cultures, a nurse practitioner has observed various practices that seemed unfamiliar. The nurse practitioner has learned that cultural rituals and practices during pregnancy seem to have one purpose in common. Which statement best describes that purpose? A.) To promote family unity B.) To ward off the evil eye C.) To appease the gods of fertility D.) To protect the mother and fetus during pregnancy

D

In planning for the care of a 30-year-old woman with pregestational diabetes, the nurse recognizes that the most important factor affecting pregnancy outcome is the: A.) Mother's age B.) Number of years since diabetes was diagnosed C.) Amount of insulin required prenatally D.) Degree of glycemic control during pregnancy

D

In which condition is the pregnant client at risk for having higher-than-normal levels of human chorionic gonadotropin (hCG), excessive vomiting and mild vaginal bleeding? A.) Miscarriage B.) Ectopic pregnancy C.) Intrauterine growth restriction D.) Gestational trophoblastic disease

D

Many pregnant women have questions regarding work and travel during pregnancy. Which education is a priority for the nurse to provide? A.) Women should sit for as long as possible and cross their legs at the knees from time to time for exercise B.) Women should avoid seat belts and shoulder restraints in the car because they press on the fetus C.) Metal detectors at airport security checkpoints can harm the fetus if the woman passes through them a number of times D.) While working or traveling in a car or on an airplane, women should arrange to walk around at least every hour or so

D

Nurses should be aware of the strengths and limitations of various biochemical assessments during pregnancy. Which statement regarding monitoring techniques is the most accurate? A.) Chorionic villus sampling (CVS) is becoming more popular because it provides early diagnosis B.) MSAFP screening is recommended only for women at risk for NTDs C.) PUBS is one of the triple-marker tests for Down syndrome D.) MSAFP is a screening tool only; it identifies candidates for more definitive diagnostic procedures

D

Nurses should be cognizant of what information with regard to the noncontraceptive medical effects of combination oral contraceptives (COCs)? A.) COCs can cause TSS if the prescription is wrong B.) Hormonal withdrawal bleeding is usually a little more profuse than in normal menstruation and lasts a week for those who use COCs C.) COCs increase the risk of endometrial and ovarian cancers D.) Effectiveness of COCs can be altered by some over-the-counter medications and herbal supplements

D

Postoperative care of the pregnant woman who requires abdominal surgery for appendicitis includes which additional assessment? A.) Intake and output (I&O) and intravenous (IV) site B.) Signs and symptoms of infection C.) Vital signs and incision D.) Fetal heart rate (FHR) and uterine activity

D

Pregnancy hormones prepare the vagina for stretching during labor and birth. Which change related to the pelvic viscera should the nurse share with the client? A.) Because of a number of changes in the cervix, abnormal Papanicolaou (Pap) tests are easier to evaluate B.) Quickening is a technique of palpating the fetus to engage it in passive movement C.) The deepening color of the vaginal mucosa and cervix (Chadwick sign) usually appears in the second trimester or later as the vagina prepares to stretch during labor D.) Increased vascularity of the vagina increases sensitivity and may lead to a high degree of arousal, especially in the second trimester

D

Since the gene for cystic fibrosis was identified in 1989, data can be collected for the purposes of genetic counseling for couples regarding carrier status. According to the most recent statistics, how often does cystic fibrosis occur in Caucasian live births? A.) 1 in 100 B.) 1 in 1000 C.) 1 in 2000 D.) 1 in 3200

D

Syphilis is a complex disease that can lead to serious systemic illness and even death if left untreated. Which manifestation differentiates primary syphilis from secondary syphilis? A.) Fever, headache, and malaise B.) Widespread rash C.) Identified by serologic testing D.) Appearance of a chancre 2 months after infection

D

The major source of nutrients in the diet of a pregnant woman should be composed of what? A.) Simple sugars B.) Fats C.) Fiber D.) Complex carbohydrates

D

The nurse finds that the blood pH of a pregnant client who is diabetic is 6.5. What would the nurse administer to normalize the client's blood pH? A.) Dextrose solution B.) Normal saline solution C.) Sodium citrate solution D.) Sodium bicarbonate solution

D

The nurse is caring for a client with gestational diabetes. Which education would the nurse provide the client regarding the use of insulin? A.) Store unused vials of insulin in the freezer B.) Shake the prepared syringes well before use C.) Administer long-acting insulin before meals D.) Inject insulin in the abdomen

D

The nurse is caring for a client with insulin-dependent diabetes mellitus in the first trimester of pregnancy. The client feels dizzy and lethargic and her blood glucose level is 50 mg/dL. What is the priority nursing action in this case? A.) Ask the dietician to recommend a sugar-free diet to the client B.) Assess the client about symptoms of retinopathy and nephropathy C.) Assess serum progesterone and estrogen levels D.) Provide the client a dose of glucose gel or a glucose tablet

D

The nurse is caring for a pregnant client at 19 weeks of gestation. On reviewing the ultrasound reports, the nurse notes that the fetus has a ventricular septal defect (VSD). The nurse knows which type of ultrasound helps detect VSD? A.) Limited examination B.) Nonmedical examination C.) Standard or basic examination D.) Specialized or targeted examination

D

The nurse is caring for a pregnant client who is prescribed levothyroxine for hypothyroidism. The client is also prescribed an iron supplement. Which education would the nurse provide the client about taking these medications? A.) Take both medications together in the morning B.) Take levothyroxine 1 hour after taking the iron supplement C.) Take the iron supplement 2 hours after taking levothyroxine D.) Take the two medications at different times of the day

D

The nurse is performing a fetal acoustic stimulation test (FAST) in conjunction with a nonstress test. The nurse observes a nonreactive baseline fetal heart rate (FHR) after 5 minutes. What is the best nursing intervention in this situation? A.) Performing the test after an interval of 2 hours B.) Performing a contraction stress test immediately C.) Administering 0.5 milliunits/min oxytocin for 20 minutes D.) Activating a stimulator for 3 seconds on the patient's abdomen

D

The nurse providing care in a women's health care setting must be knowledgeable about STIs. Which STIs can be successfully treated? A.) HSV B.) AIDS C.) Venereal warts D.) Chlamydia

D

The nursing instructor is teaching nursing students about the daily fetal movement count. Which statement by a student indicates a need for further teaching? A.) "Fetal movements decrease in clients who consume alcohol" B.) "Fetal movements cannot be detected during the fetus sleep cycle C.) "Fetal movements cannot be easily detected in obese clients" D.) "Fetal movements decrease one week before the expected date of delivery"

D

What is the importance of obtaining informed consent for a number of contraceptive methods? A.) Contraception is an invasive procedure that requires hospitalization B.) The method may require a surgical procedure to insert a device C.) The contraception method chosen may be unreliable D.) The method chosen has potentially dangerous side effects

D

What represents a typical progression through the phases of a woman's establishing a relationship with the fetus? A.) Accepts the fetus as distinct from herself; accepts the biologic fact of pregnancy; has feelings of caring and responsibility B.) Fantasizes about the child's gender and personality; views the child as part of herself; becomes introspective C.) Views the child as part of herself; has feelings of well-being; accepts the biologic fact of the pregnancy D.) I am pregnant; I am going to have a baby; I am going to be a mother

D

When caring for a pregnant woman with cardiac problems, the nurse must be alert for the signs and symptoms of cardiac decompensation. Which critical findings would the nurse find on assessment of the client experiencing this condition? A.) Regular heart rate and hypertension B.) Increased urinary output, tachycardia, and dry cough C.) Shortness of breath, bradycardia, and hypertension D.) Dyspnea, crackles, and an irregular, weak pulse

D

Which action is the first priority for the nurse who is assessing the influence of culture on a client's diet? A.) Evaluate the client's weight gain during pregnancy B.) Assess the socioeconomic status of the client C.) Discuss the four food groups with the client D.) Identify the food preferences and methods of food preparation common to the clients culture

D

Which assessment by the client indicates the need for further teaching about palpating the cervix to assess changes that indicate ovulation? A.) "The cervical os becomes slightly dilated during ovulation" B.) "The cervical mucus is watery and clear during ovulation" C.) "The cervix softens and rises in the vagina during ovulation" D.) "The cervical mucus is copious and thick during ovulation"

D

Which assessment finding is most concerning for a client who is at 38 weeks of gestation and is having difficulty catching her breath, even when sitting? A.) Ansarca B.) Irregular heartbeat C.) Displaced point of maximum impulse (PMI) D.) 5-pound weight gain over the past week

D

Which assessment finding would the nurse recognize as an indicator for early screening for gestational diabetes mellitus (GDM)? A.) The client is 24 years of age B.) The client's body mass index (BMI) if 22 C.) The client does not have diabetes D.) The client had a previous stillbirth

D

Which client is a safe candidate for the use of oral contraceptives? A.) 39 year old client with a history of thrombophlebitis B.) 16 year old client with a benign liver tumor C.) 20 year old client who suspects she may be pregnant D.) 43 year old client who does not smoke cigarettes

D

Which complications are increased in pregnant clients with gonococcal infection? A.) Condylomata lata B.) Tuboovarian abscess C.) Chlamydial infection D.) Amniotic infection syndrome

D

Which condition describes the pregnant client with severe and persistent vomiting who has lost weight, is dehydrated and has electrolyte abnormalities? A.) Tetany B.) Glossitis C.) Hypocalcemia D.) Hyperemesis gravidarum

D

Which condition is indicated when a pregnant client's fundal height has not changed in the past 4 weeks? A.) Polyhydramnios B.) Multifetal gestation C.) Maternal malnourishment D.) Intrauterine growth restriction (IUGR)

D

Which food is a common protein source for a Mexican client who is pregnant? A.) Veal B.) Mussels C.) Dal (lentils) D.) Chorizo (sausage)

D

Which gastrointestinal alteration of pregnancy is a normal finding? A.) Insufficient salivation (ptyalism) is caused by increases in estrogen B.) Acid indigestion (pyrosis) begins early but declines throughout pregnancy C.) Hyperthyroidism often develops (temporarily) because hormone production increases D.) Nausea and vomiting rarely have harmful effects on the fetus and may be beneficial

D

Which hormone is essential for maintaining pregnancy? A.) Estrogen B.) hCG C.) Oxytocin D.) Progesterone

D

Which laboratory finding indicates that the female client is pregnant? A.) Decreased level of insulin B.) Increased levels of tyroxine C.) Increased levels of follicle stimulating hormone (FSH) D.) Increased levels of human chorionic gonadotropin (hCG)

D

Which laboratory findings indicate anemia in the pregnant client reporting dizziness and fatigue? A.) Hematocrit value of 35% B.) Hematocrit value of 40% C.) Hemoglobin value of 11 g/dL D.) Hemoglobin value of 10 g/dL

D

Which low-level finding would indicate that the client has an ectopic pregnancy? A.) Insulin B.) Anemia C.) Thrombocytopenia D.) Human chorionic gonadotropin (hCG)

D

Which medication is ideal for the treatment of systemic lupus erythematosus (SLE) in a pregnant client? A.) Aspirin B.) Azathioprine C.) Prednisone D.) Hydroxychloroquine

D

Which medication is recommended treatment for the prevention of human immunodeficiency virus (HIV) transmission to the fetus during pregnancy? A.) Acyclovir B.) Ofloxacin C.) Podophyllin D.) Zidovudine

D

Which medication would the primary health care provider ask the nurse to administer to a client who has a history of a myocardial infarction (MI) and is having a vaginal delivery? A.) Oxytocin B.) Diuretics C.) Anticoagulant D.) Epidural analgesia

D

Which nursing advice is appropriate for the 14-week pregnant client asks if it is safe to have a drink with dinner now that her first trimester is complete? A.) "Because you're in your second trimester, there's no problem with having one drink with dinner" B.) "One drink every night is too much. One drink three times a week should be fine" C.) "Because you're in your second trimester, you can drink as much as you like" D.) "Because no one knows how much or how little alcohol it takes to cause fetal problems, the best course is to abstain throughout your pregnancy"

D

Which nursing advice is appropriate for the pregnant client who wants to have a nurse-midwife provide obstetric care? A.) She will have to give birth at home B.) She must see an obstetrician and the midwife during pregnancy C.) She will not be able to have epidural analgesia for labor pain D.) She must be having a low-risk pregnancy

D

Which nursing information is appropriate for a client who is prescribed a single-rod etonogestrel implant (Implanon) for contraception and believes she can have multiple sexual partners now? A.) "Your partners will have to use a condom lubricated with N-9" B.) "Encourage your partners to use a natural skin condom" C.) "It is better to use a condom, because there may be a risk for pregnancy" D.) "You must avoid multiple sexual partners and use a condom to prevent sexually transmitted infections"

D

Which nursing information is appropriate when reassuring and educating the pregnant client about changes in her blood pressure? A.) A blood pressure cuff that is too small produces a reading that is too low; a cuff that is too large produces a reading that is too high B.) Shifting the client's position and changing from arm to arm for different measurements produces the most accurate composite blood pressure reading at each visit C.) The systolic blood pressure increases slightly as pregnancy advances; the diastolic pressure remains constant D.) Compression of the iliac veins and inferior vena cava by the uterus contributes to hemorrhoids in the latter stage of term pregnancy

D

Which nursing instruction is appropriate when counseling a woman about getting enough iron in her diet? A.) Milk, coffee and tea aid iron absorption if consumed at the same time as iron B.) Iron absorption is inhibited by a diet rich in vitamin C C.) Iron supplementation are permissible for children in small doses D.) Constipation is common with iron supplements

D

Which nursing intervention reduces urinary frequency in the pregnant client? A.) Advise the client to limit fluid intake B.) Encourage the client to wear a perineal pad C.) Advise the client to empty her bladder regularly D.) Encourage the client to perform Kegel exercises

D

Which nursing report is appropriate for an expectant couple that asks about intercourse during pregnancy and if it is safe for the baby? A.) Intercourse should be avoided if any spotting from the vagina occurs afterward B.) Intercourse is safe until the third trimester C.) Safer-sex practices should be used once the membranes rupture D.) Intercourse and orgasm are often contraindicated if a history or signs of preterm labor are present

D

Which nursing response is appropriate for a female client who conceived in the first month after discontinuing combined oral contraceptives (COCs) and is worried about the effect of possible COC use on this pregnancy? A.) "The child may have a birth defect" B.) "Conception is safe only after 6 months of discontinuing the pill" C.) "The child may have iron-deficiency anemia" D.) "There is no evidence that COCs cause maternal or fetal harm"

D

Which nursing statement is appropriate for a married couple discussing male and female sterilization? A.) "Male and female sterilization are 100% effective" B.) "A vasectomy may have a slight effect on sexual performance" C.) "Tubal ligation can be easily reserved if you change your mind in the future" D.) "Major complications after sterilization are rare"

D

Which relevant drug history is appropriate for the client who had a dark complexion with brownish pigmentation over the cheeks, nose and forehead during pregnancy, which has faded and has recurred? A.) Antibiotics B.) Antipsoriatics C.) Antihistamines D.) Contraceptives

D

Which represents a positive sign of pregnancy? A.) Morning sickness B.) Quickening C.) Positive pregnancy test D.) Fetal heartbeat auscultated with Doppler/fetoscope

D

Which statement by the student nurse regarding the advantages of fertility awareness-based methods (FABs) of contraception indicates effective learning? A.) FABs protect against sexually transmitted infections (STIs) B.) There is no difficulty with adherence to this contraceptive method C.) There is increased effectiveness in women with irregular cycles D.) FABs promote increased involvement and intimacy with the partner

D

Which statement mades by the pregnant client indicates that she and her husband are adapting to the pregnancy? A.) "I am still in shock that I am pregnant" B.) "I'm not sure that I'm ready for parenthood" C.) "My husband is unsure of everything and is distant" D.) "I plan to take an extended leave of absence after the birth"

D

Which statement regarding the term contraceptive failure rate is the most accurate? A.) The contraceptive failure rate refers to the percentage of users expected to have an accidental pregnancy over a 5-year span B.) It refers to the minimum rate that must be achieved to receive a government license C.) The contraceptive failure rate increases over time as couples become more careless D.) It varies from couple to couple, depending on the method and the users

D

Which strengths and limitations of various biochemical assessments during pregnancy should the nurse be aware of? A.) Chorionic villus sampling (CVS) is becoming more popular because it provides early diagnosis B.) Screening for maternal serum alpha-fetoprotein (MSAFP) levels is recommended only for women at risk for neural tube defects C.) Percutaneous umbilical blood sampling (PUBS) is one of the quad-screen tests for Down syndrome D.) MSAFP is a screening tool only; it identifies candidates for more definitive procedures

D

While assessing the vital signs of a pregnant woman in her third trimester, the client complains of feeling faint, dizzy, and agitated. Which nursing intervention is appropriate? A.) Have the patient stand up, and then retake her BP B.) Have the patient sit down, and then hold her arm in a dependent position C.) Have the patient lie supine for 5 minutes, and then recheck her BP on both arms D.) Have the patient turn to her left side, and then recheck her BP in 5 minutes

D

While performing a contraction stress test in a pregnant client, the nurse finds that the client has three uterine contractions in a 10-minute period with no significant variable decelerations. The nurse communicates the test findings to the primary health care provider, and which instruction would the nurse expect to receive? A.) "Repeat the test in the client the next day" B.) "Administer intravenous fluids to the client" C.) "Immediately admit the client to the hospital" D.) "Resume a routine weekly testing schedule for the client"

D

hCG is an important biochemical marker for pregnancy and therefore the basis for many tests. Which statement regarding hCG is true? A.) hCG can be detected as early as weeks after conception B.) hCG levels gradually and uniformly increase throughout pregnancy C.) Significantly lower-than-normal increases in the levels of hCG may indicate a postdate pregnancy D.) Higher-than-normal levels of hCG may indicate an ectopic pregnancy or Down syndrome

D

Which are the long-acting reversible contraceptive methods (select all that apply): A.) Condoms B.) Diaphragm C.) Spermicides D.) Contraceptive implants E.) Intrauterine device (IUD)

D, E

Which medications can cause neural tube defects in the fetus (select all that apply): A.) Aspirin B.) Folic acid supplements C.) Pyridoxine supplements D.) Valproic acid E.) Carbamazepine

D, E

A client is undergoing percutaneous umbilical blood sampling (PUBS). What is the best intervention for the nurse to perform after conducting the test? A.) Monitor the fetal heart rate (FHR) B.) Give fluids to the client frequently C.) Elevate the client's bed to a 60 degree angle D.) Check the patient's blood glucose levels

A

On vaginal examination of a 30-year-old woman, the nurse documents the following findings: profuse, thin, grayish-white vaginal discharge with a fishy odor and complaints of pruritus. Based upon these findings, which condition would the nurse suspect? A.) Bacterial vaginosis B.) Candidiasis C.) Trichomoniasis D.) Gonorrhea

A

Screening at 24 weeks of gestation reveals that a pregnant woman has gestational diabetes mellitus (GDM). In planning her care, the nurse and the client mutually agree that an expected outcome is to prevent injury to the fetus as a result of GDM. This fetus is at the greatest risk for which condition? A.) Macrosomia B.) Congenital anomalies of the central nervous system C.) Preterm birth D.) Low birth weight

A

The blood glucose level of a pregnant client is 325 mg/dL. Which test would be performed on the client to assess the risk of maternal or intrauterine fetal death? A.) Ketones in urine B.) Glucose in urine C.) Arterial blood gases D.) Abdominal ultrasonography

A

What is the correct term used to describe the mucous plug that forms in the endocervical canal? A.) Operculum B.) Leukorrhea C.) Funic souffle D.) Ballottement

A

Dehydration may increase which risk in the client in the ninth month of pregnancy (select all that apply): A.) Cramping B.) Contractions C.) Preterm labor D.) Fetal neurotoxicity E.) Physiologic anemia

A, B, C

Which congenital anomalies can occur as a result of the use of antiepileptic drugs (AEDs) in pregnancy (select all that apply): A.) Cleft lip B.) Congenital heart disease C.) Neural tube defects D.) Gastroschisis E.) Diaphragmatic hernia

A, B, C

Which sexually transmitted infections are caused by bacteria (select all that apply): A.) Chlamydia B.) Gonorrhea C.) Trichomoniasis D.) Hepatitis A and B E.) Lymphogranuloma venereum

A, B, E

A 3-year-old girls mother is 6 months pregnant. What concern is this child most likely to verbalize? A.) How the baby will get out? B.) How will the baby eat? C.) Will you die having the baby? D.) What color eyes will the baby have?

B

A client with maternal phenylketonuria (PKU) has come to the obstetrical clinic to begin prenatal care. Why would this preexisting condition result in the need for closer monitoring during pregnancy? A.) PKU is a recognized cause of preterm labor B.) The fetus may develop neurologic problems C.) A pregnant woman is more likely to die without strict dietary control D.) Women with PKU are usually mentally handicapped and should not reproduce

B

A patient in her first trimester complains of nausea and vomiting. She asks, Why does this happen? What is the nurse's best response? A.) Nausea and vomiting are due to an increase in gastric motility B.) Nausea and vomiting may be due to changes in hormones C.) Nausea and vomiting are related to an increase in glucose levels D.) Nausea and vomiting are caused by a decrease in gastric secretions

B

Diabetes in pregnancy puts the fetus at risk in several ways. Nurses should be aware of which factor? A.) With good control of maternal glucose levels, sudden and unexplained stillbirth is no longer a major concern B.) The most important of perinatal loss in diabetic pregnancy is congenital malformations C.) Infants of mothers with diabetes have the same risk for respiratory distress syndrome because of the careful monitoring D.) At birth, the neonate of a diabetic mother is no longer at any greater risk

B

Ideally, when should prenatal care begin? A.) Before the first missed menstrual period B.) After the first missed menstrual period C.) After the second missed menstrual period D.) After the third missed menstrual period

B

The client makes an appointment for preconception counseling. The woman has a known heart condition and is unsure if she should become pregnant. Which is the only cardiac condition that would cause concern? A.) Marfan syndrome B.) Eisenmenger syndrome C.) Heart transplant D.) Ventricular septal defect (VSD)

B

The nurse is caring for a client in the first trimester of pregnancy who is prescribed propylthiouracil (PTU) for hyperthyroidism. What are the fetal side effects of this medication? A.) Facial anomalies B.) Hepatic toxicity C.) Esophageal atresia D.) Developmental delay

B

Which action is the highest priority for the nurse when educating a pregnant adolescent? A.) Emphasize the need to eliminate common teenage snack foods because they are high in fat and sodium B.) Determine the weight gain needed to meet adolescent growth, and add 35 pounds C.) Suggest that she not eat at fast-food restaurants to avoid foods of poor nutritional value D.) Realize that most adolescents are unwilling to make dietary changes during pregnancy

B

Which area of concern is appropriate to focus on when providing care to pregnant adolescents? A.) Feelings of isolation B.) Late entry into prenatal care C.) Increased risk for cesarean delivery D.) Risk for chromosomal abnormalities

B

Which benefit regarding FAMs makes it an appealing choice for some women? A.) Adherence to strict recordkeeping B.) Absence of chemicals and hormones C.) Decreased involvement and intimacy of partner D.) Increased spontaneity of coitus

B

Which is a positive sign of pregnancy when teaching a client about the presumptive, probable and positive signs of pregnancy? A.) A positive pregnancy test B.) Fetal movements palpated by the nurse-midwife C.) Braxton hicks contractions D.) Quickening

B

Which is an emergency contraceptive that can be used by a female client after unprotected sex? A.) Oral progestin (minipill) B.) ParaGard Copper T 380A C.) Essure system D.) Etonogestrel implant

B

Which laboratory parameter requires monitoring if a pregnant client reports, "My body shook for a while when I was sitting on my couch?" A.) Blood glucose levels B.) Blood pressure C.) Complete blood cell count D.) Electroencephalogram (EEG)

B

Which laboratory testing is used to detect the human immunodeficiency virus (HIV)? A.) HIV screening B.) HIV antibody testing C.) Cluster of differentiation 4 (CD4) counts D.) Cluster of differentiation 8 (CD8) counts

B

Which nursing action is appropriate for the client at 16 weeks of gestation based on evaluation of the laboratory results - Factor VII: 135 - Factor VIII: 160 - Factor IX: 155 - Factor X: 145 - Factor XI: 130 A.) Make no changes to the plan of acre B.) Take initiatives to reduce venous thromboembolism (VTE) risk C.) Initiate factor replacement therapy D.) Immediately implement bleeding precautions

B

Which nursing information is included when discussing breast care with a pregnant client who would like to breastfeed? A.) Remove nipple jewelry before deliver B.) Avoid the use of soap when washing the nipples C.) Use a pad of synthetic liner on the back to absorb nipple leakage D.) Apply a breast shield a few weeks before delivery

B

Which nursing instruction is appropriate for a 35 year old client taking combined oral contraceptives (COCs)? A.) "COCs can cause early menopause" B.) "Avoid herbal supplements" C.) "There is a risk for iron-deficiency anemia with COCs" D.) "There is a risk for increased menstrual blood loss"

B

Which renal system adaptation is an anticipated anatomic change of pregnancy? A.) Increased urinary output makes pregnant women less susceptible to urinary infections B.) Increased bladder sensitivity and then compression of the bladder by the enlarging uterus result in the urge to urinate even when the bladder is almost empty C.) Renal (kidney) function is more efficient when the woman assumes a supine position D.) Using diuretic agents during pregnancy can help keep kidney function regular

B

Which statement by the student nurse indicated effective learning about pelvic inflammatory disease (PID)? A.) PID causes miscarriage B.) The menstrual period facilitates the development of PID C.) Neisseria gonorrhoeae is the only organism that causes PID D.) PID occurs as organisms spread from the upper genital tract in the vagina

B

Which statement by the student nurse indicates the need for further teaching about the procedure for measuring blood pressure (BP) in pregnant clients? A.) The two readings should be taken at least 1 minute apart B.) The cuff should cover approximately one-half on the upper arm C.) BP should be measured with the woman sitting or in a semi-reclining position D.) If BP is elevated, the woman should rest for at least 10 minutes before I retake her BP

B

Which statement made by a lactating woman leads the nurse to believe that the client might have lactose intolerance? A.) I always have heartburn after I drink milk B.) If I drink more than a cup of milk, I usually have abdominal cramps and bloating C.) Drinking milk usually makes me break out in hives D.) Sometimes I notice that I have bad breath after I drink a cup of milk

B

Which statement regarding multifetal pregnancy is incorrect? A.) The expectant mother often develops anemia because the fetuses have a greater demand for iron B.) Twin pregnancies come to term with the same frequency as single pregnancies C.) The mother should be counseled to increase her nutritional intake and gain more weight D.) Backache and varicose veins often are more pronounced with a multifetal pregnancy

B

Which type of cervical mucus would you expect if a fertile woman is in postovulation? A.) Scant B.) Thick, cloudy and sticky C.) Clear, wet, sticky and slippery D.) Cloudy, yellow or white, and sticky

B

Which type of cultural concern is the most likely deterrent to many women seeking prenatal care? A.) Religion B.) Modesty C.) Ignorance D.) Belief that physicians are evil

B

Which viral sexually transmitted infection (STI) is the most prevalent in the United States? A.) Herpes simplex virus type 2 (HSV-2) B.) Human papillomavirus (HPV) C.) Human immunodeficiency virus (HIV) D.) Cytomegalovirus (CMV)

B

Within how many days can a client use emergency contraception to prevent pregnancy after unprotected sex? A.) 10 days B.) 3 days C.) 12 days D.) 21 days

B

A 30-year-old gravida 3, para 2-0-0-2 is at 18 weeks of gestation. Which screening test should the nurse recommend be ordered for this client? A.) BPP B.) Chorionic villi sampling C.) MSAFP screening D.) Screening for diabetes mellitus

C

The nurse is providing education to a client regarding the normal changes of the breasts during pregnancy. Which statement regarding these changes is correct? A.) The visibility of blood vessels that form an intertwining blue network indicates full function of the Montgomery tubercles and possibly an infection of the tubercles B.) The mammary glands do not develop until 2 weeks before labor C.) Lactation is inhibited until the estrogen level declines after birth D.) Colostrum is the yellowish oily substance used to lubricate the nipples for breastfeeding

C

To manage her diabetes appropriately and to ensure a good fetal outcome, how would the pregnant woman with diabetes alter her diet? A.) Eat six small equal meals per day B.) Reduce the carbohydrates in her diet C.) Eat her meals and snacks on a fixed schedule D.) Increase her consumption of protein

C

Which organism is the causative agent for ophthalmia neonatorum? A.) Neisseria gonorrhoeae B.) Human papillomavirus C.) Chlamydia trachomatis D.) Gardnerella and Mobiluncus

C

During pregnancy, many changes occur as a direct result of the presence of the fetus. Which of these adaptations meet this criterion (select all that apply): A.) Leukorrhea B.) Development of the operculum C.) Quickening D.) Ballottement E.) Lightening

C, D, E

The nurse is caring for a pregnant client with rheumatic heart disease and activity intolerance. Which nursing interventions would the nurse perform to reduce activity intolerance (select all that apply): A.) Monitor fluid intake and output B.) Teach the client to lie on her side C.) Advise the client to maintain an activity log D.) Encourage the client to reduce activity that cause fatigue E.) Help the client develop an individualized program of activity and rest

C, D, E

A 40 year old woman with a body mass index (BMI) over 30 is 10 weeks pregnant. Which diagnostic tool is appropriate to suggest to her at this time? A.) Biophysical profile B.) Amniocentesis C.) Maternal serum alpha-fetoprotein (MSAFP) D.) Transvaginal ultrasound

D

A client currently uses a diaphragm and spermicide for contraception. She asks the nurse to explain the major differences between the cervical cap and the diaphragm. What is the most appropriate response by the nurse? A.) No spermicide is used with the cervical cap, so its less messy B.) The diaphragm can be left in place longer after intercourse C.) Repeated intercourse with the diaphragm is more convenient D.) The cervical cap can be safely used for repeated acts of intercourse without adding more spermicide later

D

A client is using a basal body temperature (BBT) method of contraception. She calls the clinic and tells the nurse, "My period is due in a few days, and my temperature has not gone up." Which is the most appropriate response? A.) "This probably means you're pregnant" B.) "Don't worry; it's probably nothing" C.) "Have you been ill this month?" D.) "You probably didn't' ovulate during this cycle"

D

Numerous changes in the integumentary system occur during pregnancy. Which change persists after birth? A.) Epulis B.) Chloasma C.) Telangiectasia D.) Striae gravidarum

D

Nutrition is an alterable and important preventive measure for a variety of potential problems such as low birth weight and prematurity. While completing the physical assessment of the pregnant client, the nurse is able to evaluate the clients nutritional status by observing a number of physical signs. Which physical sign indicates to the nurse that the client has unmet nutritional needs? A.) Normal heart rate, rhythm, and blood pressure B.) Bright, clear, and shiny eyes C.) Alert and responsive with good endurance D.) Edema, tender calves, and tingling

D

To reassure and educate their pregnant clients regarding changes in their blood pressure, nurses should be cognizant of what? A.) A blood pressure cuff that is too small produces a reading that is too low; a cuff that is too large produces a reading that is too high B.) Shifting the client's position and changing from arm to arm for different measurements produces the most accurate composite blood pressure reading at each visit C.) Systolic blood pressure slightly increases as the pregnancy advances; diastolic pressure remains constant D.) Compression of the iliac veins and inferior vena cava by the uterus contributes to hemorrhoids in the later stage of a term pregnancy

D

What form of heart disease in women of childbearing years generally has a benign effect on pregnancy? A.) Cardiomyopathy B.) Rheumatic heart disease C.) Congenital heart disease D.) Mitral valve prolapse

D

Which information can be interpreted from the nurse observing inverted nipples on the breasts of a pregnant client? A.) The client had breast reduction surgery in the past B.) The client applied iodine tincture on the nipples C.) These are normal breast changes during pregnancy D.) The client's baby will probably have difficulty latching

D

Which sign or symptom is considered a first-trimester warning sign and should be immediately reported by the pregnant woman to her health care provider? A.) Nausea with occasional vomiting B.) Fatigue C.) Urinary frequency D.) Vaginal bleeding

D

Which statement by the client indicates the need for further teaching when providing exercise tips to an 18-week pregnant client? A.) I should exercise regularly for 30 minutes at a time B.) I should decrease weight-bearing exercises C.) I should take my pulse for 10 minutes after exercising D.) I should lie on my back for 10 minutes after exercising

D

Which statement by the pregnant client indicates the need for further teaching about body mechanics to decrease discomfort related to the lumbar curve of pregnancy? A.) "I should avoid platform shoes and high heels" B.) "I should use a pillow in the car to support my lower back area" C.) "I should change positions often if I have to stand for a long time" D.) "I should adjust my car seat such that my knees are lower than my hips"

D

Which tool is the most sensitive assessment for the diagnosis of placental abruption after abdominal trauma? A.) Ultrasonography B.) Kleihauer-Betke assay C.) Abdominal palpation D.) External fetal monitoring

D

Which sign of pregnancy may manifest as an increase of urinary frequency in the client? A.) Hegar sign B.) Goodell sign C.) Ballottement D.) Chadwick sign

A

What are signs and symptoms of supine hypotension (select all that apply): A.) Pallor B.) Acid reflux C.) Dizziness D.) Tachycardia E.) Breathlessness F.) Dry, warm skin

A, C, D, E

Which fetal impairment is the nurse trying to prevent by recommending the pregnant client avoid fish such as swordfish, tilefish and king mackrel? A.) Bone development B.) Protein metabolism C.) Hemoglobin formation D.) Neurologic development

D

The nurse is caring for a client with rheumatic heart disease (RHD). Which medication would the primary health care provider prescribe to prevent pulmonary edema? A.) Verapamil B.) Furosemide C.) Atenolol D.) Warfarin

B

Which finding in the urinalysis of a pregnant woman is considered a variation of normal? A.) Proteinuria B.) Glycosuria C.) Bacteria in the urine D.) Ketones in the urine

B

Which factors change significantly during pregnancy (select all that apply): A.) Bilirubin levels B.) Albumin levels C.) Platelet count D.) Hematocrit value E.) Gastric secretions

B, D, E

Pregnant adolescents are at greater risk for decreased BMI and fad dieting with which condition? A.) Obesity B.) Gestational diabetes C.) Low-birth-weight babies D.) High-birth-weight babies

C

Which mineral intake is restricted in a pregnant client with renal failure? A.) Zinc B.) Iron C.) Sodium D.) Manganese

C

Which micronutrients can be obtained through dietary sources to meet the nutritional needs of a pregnant client (select all that apply): A.) Iron B.) Folate C.) Calcium D.) Vitamin D E.) Vitamin B12

C, D, E

When assessing a pregnant client, the nurse is aware of which complications associated with polyhydramnios (select all that apply): A.) Ketoacidosis B.) Placental abruption C.) Uterine dysfunction D.) Gestational diabetes E.) Postpartum hemorrhage

B, C, E

A pregnant woman reports that she is still playing tennis at 32 weeks of gestation. Which recommendation would the nurse make for this particular client after a tennis match? A.) Drink several glasses of fluid B.) Eat extra protein sources such as peanut butter C.) Enjoy salty foods to replace lost sodium D.) Consume easily digested sources of carbohydrate

A

A client who is pregnant already has type 2 diabetes and a hemoglobin A1c of 7. Which client condition would the nurse use to categorize this client as a diabetic? A.) Gestational diabetes B.) Insulin-dependent diabetes complicated by pregnancy C.) Pregestational diabetes mellitus D.) Non-insulin-dependent diabetes with complications

C

A pregnant woman's BPP score is 8. She asks the nurse to explain the results. How should the nurse respond at this time? A.) The test results are within normal limits B.) Immediate delivery by cesarean birth is being considered C.) Further testing will be performed to determine the meaning of this score D.) An obstetric specialist will evaluate the results of this profile and, within the next week, will inform you of your options regarding delivery

A

While working with the pregnant client in her first trimester, what information does the nurse provide regarding when CVS can be performed (in weeks of gestation)? A.) 4 B.) 8 C.) 10 D.) 14

C

Which is a common maternal complication of Chlamydia? A.) Meningitis B.) Preterm labor C.) Chorioamnionitis D.) Postpartum sepsis

B

Which statement made by the client correlates with a probable pregnancy? A.) My period is three weeks late B.) I can feel the baby moving around C.) I can feel my uterine contract, but it does not hurt D.) I have been experiencing nausea and vomiting almost daily

C

How does the nurse document a NST during which two or more FHR accelerations of 15 beats per minute or more occur with fetal movement in a 20-minute period? A.) Non-reactive B.) Positive C.) Negative D.) Reactive

D

Which diet is recommended for the client who is 6 months pregnant and diagnosed with diverticulosis? A.) Whole grains, bran, vegetables and fruits B.) Citrus fruits and dark green leafy vegetables C.) Shellfish, liver, meats, whole grains and milk D.) Iodized salt, seafood, milk products and rolls

A

Which STI does not respond well to antibiotic therapy? A.) Chlamydia B.) Gonorrhea C.) Genital herpes D.) Syphilis

C

Which clinical finding in a primiparous client at 32 weeks of gestation might be an indication of anemia? A.) Ptyalism B.) Pyrosis C.) Pica D.) Decreased peristalsis

C

Which clinical significance does a maternal blood Coombs test with a titer of 1:8 and increasing indicate? A.) Fetal lung maturity B.) Significant Rh compatability C.) Significant Rh incompatability D.) Fetus with trisomy 13, 18 or 21

C

Which information is an important consideration when comparing the CST with the NST? A.) The NST has no known contraindications B.) The CST has fewer false-positive results when compared with the NST C.) The CST is more sensitive in detecting fetal compromise, as opposed to the NST D.) The CST is slightly more expensive than the NST

A

Which condition is the most life-threatening virus to the fetus and neonate? A.) Hepatitis A virus (HAV) B.) Herpes simplex virus (HSV) C.) Hepatitis B virus (HBV) D.) Cytomegalovirus (CMV)

C

The nurse is informing the pregnant client with a cardiac disorder about the dietary changes that are needed. Which instructions would the nurse include in the teaching (select all that apply): A.) "Take iron and folic acid supplements daily" B.) "Increase your daily intake of dietary fiber" C.) "Take a stool softener daily as prescribed" D.) "Cut intake of dark green leafy vegetables" E.) "Include potassium-rich foods in the diet"

A, B, C, E

Transvaginal ultrasonography is often performed during the first trimester. While preparing a 6-week gestational client for this procedure, she expresses concerns over the necessity for this test. The nurse should explain that this diagnostic test may be indicated for which situations (select all that apply): A.) Multifetal gestation B.) Obesity C.) Fetal abnormalities D.) Amniotic fluid volume E.) Ectopic pregnancy

A, B, C, E

Which condition would affect a client at 10 weeks of gestation whose breasts have been leaking colostrum? A.) Elevated prolactin level B.) Hypersecretion of hormones C.) Inaccurate gestational age D.) Decreased human placental lactogen level

C

Which laboratory testing is anticipated on the initial prenatal visit for a client who is at 8 weeks of gestation (select all that apply): A.) Pap test B.) Urine culture C.) Rubella titer D.) 1-hour glucose tolerance test E.) Vaginal and anal culture for group B Streptococcus (GBS) F.) Gonorrhea and chlamydia cervical cultures

A, B, C, E

In terms of the incidence and classification of diabetes, which information should the nurse keep in mind when evaluating clients during their ongoing prenatal appointments? A.) Type 1 diabetes is most common B.) Type 2 diabetes often goes undiagnosed C.) GDM means that the woman will receive insulin treatment until 6 weeks after birth D.) Type 1 diabetes may become type 2 during pregnancy

B

A client at 34 weeks of gestation seeks guidance from the nurse regarding personal hygiene. Which information should the nurse provide? A.) Tub bathing is permitted even in late pregnancy unless membranes have ruptured B.) The perineum should be wiped from back to front C.) Bubble bath and bath oils are permissible because they add an extra soothing and cleansing action to the bath D.) Expectant mothers should use specially treated soap to cleanse the nipples

A

A client exhibits a thick, white, lumpy, cottage cheeselike discharge, along with white patches on her labia and in her vagina. She complains of intense pruritus. Which medication should the nurse practitioner order to treat this condition? A.) Fluconazole B.) Tetracycline C.) Clindamycin D.) Acyclovir

A

Which behavioral change would the nurse observe in the spouse of a pregnant client during the focusing phase? A.) Engages in building a relationship with the newborn B.) Engages in discussions with others about the philosophy of life C.) Engages in extramarital affairs because of a lack of partner's attention D.) Has difficulty accepting changes in life plans and lifestyles

A

A client asks her nurse, "My doctor told me that he is concerned with the grade of my placenta because I am overdue. What does that mean?" What is the nurse's best response? A.) Your placenta changes as your pregnancy progresses, and it is given a score that indicates how well it is functioning B.) Your placenta isn't working properly, and your baby is in danger C.) We need to perform an amniocentesis to detect if you have any placental damage D.) Dont worry about it. Everything is fine

A

Which client condition would the nurse relate to hypoglycemia in a diabetic client? A.) Clammy skin B.) Rapid breathing C.) Nausea or vomiting D.) Increased urination

A

Which is the client's first day of the last menstrual period (LMP) if the estimated date of birth (EDB) is December 2, 2019? A.) February 25, 2019 B.) March 25, 2019 C.) February 2, 2019 D.) March 2, 2019

A

Which nursing instructions are appropriate for the pregnant client about relieving constipation (select all that apply): A.) Consume at least 28 g of fiber per day B.) Eat more eggs daily C.) Eat whole grains and fresh fruits D.) Eat a good quantity of meat daily E.) Drink at least 50 mL/kg/day of fluids

A, C, E

A client states that she plans to breastfeed her newborn infant. What guidance would be useful for this new mother? A.) The mothers intake of vitamin C, zinc, and protein can now be lower than during pregnancy B.) Caffeine consumed by the mother accumulates in the infant, who may be unusually active and wakeful C.) Critical iron and folic acid levels must be maintained D.) Lactating women can go back to their prepregnant caloric intake

B

Which newborn conditions can result from taking vitamin A supplements in higher doses than prescribed (select all that apply): A.) Tetany B.) Anemia C.) Spina bifida D.) Cleft palate E.) Hypocalcemia

C, D

A nurse is teaching a pregnant client with a history of psoriasis. Which information would the nurse provide to the client? A.) Pregnancy has no effect on psoriasis B.) Staying out of direct sunlight will stop progression of disease C.) Psoriasis typically worsens during pregnant in approximately 50% of clients D.) Clients who have psoriasis during pregnancy experience a varied response

D

A client who is an intravenous drug (IV) presents with fever and complains of arthritis, nausea, vomiting and mild abdominal pain. Which additional assessment is a priority for this client? A.) Assess if the client consumes alcohol B.) Assess if the client has had any vaccinations C.) Assess for signs of dehydration and malnutrition D.) Assess for hepatitis B surface antigen

D

Which technology to test for human chorionic gonadotropin (hCG) is used in over-the-counter (OCT) pregnancy tests? A.) Radioimmunoassay B.) Radioreceptor assay C.) Latex agglutination test D.) Enzyme-linked immunosorbent assay (ELISA)

D

A pregnant woman's diet consists almost entirely of whole grain breads and cereals, fruits, and vegetables. Which dietary requirement is the nurse most concerned about? A.) Calcium B.) Protein C.) Vitamin B12 D.) Folic acid

C

A woman at 28 weeks of gestation experiences blunt abdominal trauma as the result of a fall. The nurse must closely observe the client for what? A.) Alteration in maternal vital signs, especially blood pressure B.) Complaints of abdominal pain C.) Placental absorption D.) Hemorrhage

C

Assessment of a woman's nutritional status includes a diet history, medication regimen, physical examination, and relevant laboratory tests. Which finding might require consultation to a higher level of care? A.) Oral contraceptive use may interfere with the absorption of iron B.) Illnesses that have created nutritional deficits, such as PKU, may require nutritional care before conception C.) The womans socioeconomic status and educational level are not relevant to her examination; they are the province of the social worker D.) Testing for diabetes is the only nutrition-related laboratory test most pregnant women need

B

Bell palsy is an acute idiopathic facial paralysis, the cause for which remains unknown. Which statement regarding this condition is correct? A.) Bell palsy is the sudden development of bilateral facial weakness B.) Women with Bell palsy have an increased risk for hypertension C.) Pregnant women are affected twice as often as nonpregnant women D.) Bell palsy occurs most frequently in the first trimester

B

Cardiac output increases from 30% to 50% by the 32nd week of pregnancy. What is the rationale for this change? A.) To compensate for the decreased renal plasma flow B.) To provide adequate perfusion of the placenta C.) To eliminate metabolic wastes of the mother D.) To prevent maternal and fetal dehydration

B

A 31-year-old woman believes that she may be pregnant. She took an over-the-counter (OTC) pregnancy test 1 week ago after missing her period; the test was positive. During her assessment interview, the nurse inquires about the woman's last menstrual period and asks whether she is taking any medications. The client states that she takes medicine for epilepsy. She has been under considerable stress lately at work and has not been sleeping well. Her physical examination does not indicate that she is pregnant. She has an ultrasound scan, which confirms that she is not pregnant. What is the most likely cause of the false-positive pregnancy test result? A.) The pregnancy test was taken too early B.) Anticonvulsant medications may cause the false-positive test result C.) The woman has a fibroid tumor D.) She has been under considerable stress and has a hormone imbalance

B

A client who recently had a heart transplant with no evidence of rejection asks the nurse about the safety of conceiving a child. Which is the most accurate response by the nurse? A.) "A heart transplantation does not tolerate pregnancy" B.) "You may conceive one year after the transplant" C.) "The newborn may have congenital heart disease" D.) "You may need to terminate pregnancy at any time"

B

A male client asks the nurse why it is better to purchase condoms that are not lubricated with nonoxynol-9 (a common spermicide). Which response by the nurse is the most accurate? A.) The lubricant prevents vaginal irritation B.) Nonoxynol-9 does not provide protection against STIs as originally thought; it has also been linked to an increase in the transmission of the HIV and can cause genital lesions C.) The additional lubrication improves sex D.) Nonoxynol-9 improves penile sensitivity

B

A married couple is discussing alternatives for pregnancy prevention and has asked about fertility awareness methods (FAMs). Which response by the nurse is most appropriate? A.) They're not very effective, and it is very likely that you'll get pregnant B.) FAMs can be effective for many couples; however, they require motivation C.) These methods have a few advantages and several health risks D.) You would be much safer going on the pill and not having to worry

B

A pregnant client is infected with human immunodeficiency virus (HIV), with a viral load of 800 copies/mL at 36 weeks. The client has ruptured membranes and labor is progressing rapidly. Which order is the primary health care provider likely to make? A.) Scalp pH sampling B.) Immediate vaginal birth C.) Immediate cesarean birth D.) Use of fetal scalp electrode

B

A pregnant woman at 18 weeks of gestation calls the clinic to report that she has been experiencing occasional backaches of mild-to-moderate intensity. Which intervention should the nurse recommend? A.) Kegel exercises B.) Pelvic rock exercises C.) Softer mattress D.) Bed rest for 24 hours

B

A womans obstetric history indicates that she is pregnant for the fourth time, and all of her children from previous pregnancies are living. One was born at 39 weeks of gestation, twins were born at 34 weeks of gestation, and another child was born at 35 weeks of gestation. What is her gravidity and parity using the GTPAL system? A.) 3-1-1-1-3 B.) 4-1-2-0-4 C.) 3-0-3-0-3 D.) 4-2-1-0-3

B

An 18-year-old client who has reached 16 weeks of gestation was recently diagnosed with pregestational diabetes. She attends her centering appointment accompanied by one of her girlfriends. This young woman appears more concerned about how her pregnancy will affect her social life than her recent diagnosis of diabetes. A number of nursing diagnoses are applicable to assist in planning adequate care. What is the most appropriate diagnosis at this time? A.) Risk for injury, to the fetus related to birth trauma B.) Deficient knowledge, related to diabetic pregnancy management C.) Deficient knowledge, related to insulin administration D.) Risk for injury, to the mother related to hypoglycemia or hyperglycemia

B

An MSAFP screening indicates an elevated level of alpha-fetoprotein. The test is repeated, and again the level is reported as higher than normal. What is the next step in the assessment sequence to determine the well-being of the fetus? A.) PUBS B.) Ultrasound for fetal anomalies C.) BPP for fetal well-being D.) Amniocentesis for genetic anomalies

B

During the first trimester, which of the following changes regarding her sexual drive should a client be taught to expect? A.) Increased sexual drive, because of enlarging breasts B.) Decreased sexual drive, because of nausea and fatigue C.) No change in her sexual drive D.) Increased sexual drive, because of increased levels of female hormones

B

A client who has cystic fibrosis (CF) and a normal body mass index (BMI) is pregnant. Which weight-gain recommendation would the nurse give to this client? A.) Amount gained is not as important as maintaining a well-balanced diet B.) Between 30 to 35 pounds C.) Approximately 25 pounds D.) Between 10 and 20 pounds

C

A number of metabolic changes occur throughout pregnancy. Which physiologic adaptation of pregnancy will influence the nurses plan of care? A.) Insulin crosses the placenta to the fetus only in the first trimester, after which the fetus secretes its own B.) Women with insulin-dependent diabetes are prone to hyperglycemia during the first trimester because they are consuming more sugar C.) During the second and third trimesters, pregnancy exerts a diabetogenic effect that ensures an abundant supply of glucose for the fetus D.) Maternal insulin requirements steadily decline during pregnancy

C

A nurse providing care for the antepartum woman should understand that the contraction stress test (CST): A.) Sometimes uses vibroacoustic stimulation B.) Is an invasive test; however, contractions are stimulated C.) Is considered negative if no late decelerations are observed with the contractions D.) Is more effective than non-stress test (NST) if the membranes have already been ruptured

C

A pregnant client at 14 weeks of gestation is admitted to the hospital with a diagnosis of hyperemesis gravidarum. Which is the primary goal of her treatment at this time? A.) Rest the gastrointestinal tract by restricting all oral intake for 48 hours B.) Reduce emotional distress by encouraging the client to discuss her feelings C.) Reverse fluid, electrolyte and acid-base imbalances D.) Restore the client's ability to take and retain oral fluid and foods

C

A pregnant client in the third trimester is advised to undergo an antiphospholipid antibody test. The nurse knows that which is the objective of this test? A.) To diagnose Marfan syndrome B.) To diagnose mitral valve stenosis C.) To diagnose myocardial infarction D.) To diagnose pulmonary hypertension

C

A pregnant woman at 10 weeks of gestation jogs three or four times per week. She is concerned about the effect of the exercise on the fetus. Which guidance should the nurse provide? A.) You don't need to modify your exercising any time during your pregnancy B.) Stop exercising because it will harm the fetus C.) You may find that you need to modify your exercise to walking later in your pregnancy, around the seventh month D.) Jogging is too hard on your joints; switch to walking now

C

A pregnant woman at term is transported to the emergency department (ED) after a severe vehicular accident. The obstetric nurse responds and rushes to the ED with a fetal monitor. Cardiopulmonary arrest occurs as the obstetric nurse arrives. What is the highest priority for the trauma team? A.) Obtaining IV access, and starting aggressive fluid resuscitation B.) Quickly applying the fetal monitor to determine whether the fetus viability C.) Starting cardiopulmonary resuscitation (CPR) D.) Transferring the woman to the surgical unit for an emergency cesarean delivery in case the fetus is still alive

C


Ensembles d'études connexes

Maternity and Women's Health Nursing

View Set

New Opportunities pre-intermediate Module 2-2

View Set